USMLE Microbiology Exam Practice Test

Reviewed by Editorial Team
The ProProfs editorial team is comprised of experienced subject matter experts. They've collectively created over 10,000 quizzes and lessons, serving over 100 million users. Our team includes in-house content moderators and subject matter experts, as well as a global network of rigorously trained contributors. All adhere to our comprehensive editorial guidelines, ensuring the delivery of high-quality content.
Learn about Our Editorial Process
| By Catherine Halcomb
Catherine Halcomb
Community Contributor
Quizzes Created: 1443 | Total Attempts: 6,714,021
| Attempts: 936 | Questions: 146
Please wait...
Question 1 / 146
0 %
0/100
Score 0/100
1. Eight of 10 family practice residents who had a potluck four days ago now have diarrhea with abdominal cramps, general malaise, and fever ranging from 37.5 to 38.7 C (99.5 to 101.6 F). Stools from three residents are blood tinged. Laboratory studies revealed the causal agent was a microaerophilic gram-negative, curved rod with polar flagella often in pairs to give a "seagull" appearance. It grew on special media at 42 C (107.6 F). The original contami- nation probably was found in

Explanation

The correct answer is poultry. This is because the symptoms described, such as diarrhea, abdominal cramps, and fever, are consistent with a foodborne illness. The presence of blood in the stools suggests a more severe infection. The description of the causal agent as a microaerophilic gram-negative, curved rod with polar flagella often in pairs to give a "seagull" appearance is characteristic of Campylobacter jejuni, a common cause of bacterial gastroenteritis. Poultry, especially undercooked or improperly handled chicken, is a known source of Campylobacter infection. Therefore, it is likely that the original contamination occurred in the poultry consumed at the potluck.

Submit
Please wait...
About This Quiz
USMLE Microbiology Exam Practice Test - Quiz

This USMLE Microbiology Exam Practice Test assesses key knowledge in medical microbiology, focusing on diseases, bacterial characteristics, and clinical scenarios. It prepares learners for real-world medical challenges and... see moreUSMLE certification. see less

2. A 26-year-old male presents to your office with dysuria and urethral discharge two weeks after having unprotected sexual intercourse with a new partner. Gram stain of the discharge reveals numerous neutrophils with intracellular diplococci. A sample of the discharge is placed on an antibiotic-containing medium, and bacterial colonies are cultured. This medium is best described as:

Explanation

The medium used in this scenario is described as selective. This is because it contains antibiotics, which inhibit the growth of certain bacteria while allowing the growth of others. By using this medium, the goal is to selectively promote the growth of the bacteria causing the infection, while inhibiting the growth of other bacteria that may be present in the sample. This helps in isolating and identifying the specific bacteria responsible for the patient's symptoms.

Submit
3. A 32-year-old migrant ranch worker is brought to the emergency room with painful paroxysmal involuntary muscle contractions involving mainly the jaw, neck and trunk. The causative substance has most likely traveled by which of the following routes?

Explanation

The correct answer suggests that the causative substance has most likely traveled through a wound, then through motor neuron axons, and finally reached the spinal cord. This explanation indicates that the substance entered the body through a wound and then spread through the motor neuron axons, which are responsible for transmitting signals from the central nervous system to the muscles. The fact that the muscle contractions are mainly affecting the jaw, neck, and trunk supports the idea that the substance has reached the spinal cord, which controls these muscle groups.

Submit
4. A cancer chemotherapy patient has to have her intravenous port revised after it becomes blocked and the catheter is found to contain bacterial con- taminants. Which of the following attributes is most likely to be a factor in this pathogenesis?

Explanation

Biofilm production is most likely to be a factor in the pathogenesis of the blocked intravenous port and bacterial contamination in the catheter. Biofilms are communities of bacteria that attach to surfaces and form a protective matrix. They are known to be highly resistant to antibiotics and immune responses, making them difficult to eradicate. In this case, the biofilm formed by the bacteria in the catheter could have caused the blockage and also provided a protected environment for bacterial growth, leading to contamination.

Submit
5. What is the typical means of transmission of a toxin that blocks the release of inhibitory transmitters GABA and glycine?

Explanation

A puncture wound can be a typical means of transmission for a toxin that blocks the release of inhibitory transmitters GABA and glycine. When a puncture wound occurs, it can allow the entry of toxins into the body, which can then affect the release of these inhibitory transmitters. This can disrupt the normal functioning of the nervous system and lead to various symptoms and complications. Therefore, a puncture wound can be a potential route for the transmission of such toxins.

Submit
6. A patient is admitted to the hospital because of a bleeding duodenal ulcer. Culture at 37.0 C (98.6 F) reveals a urease-positive, gram-negative, curved rod. Which of the following is a likely complication due to infection with the causal agent?

Explanation

Infection with the causal agent of a bleeding duodenal ulcer, which is a urease-positive, gram-negative, curved rod, can lead to stomach cancer as a likely complication. This is because chronic infection with certain bacteria, such as Helicobacter pylori, can cause inflammation and damage to the lining of the stomach, increasing the risk of developing stomach cancer over time.

Submit
7. A 24-year-old female presents to your office with burning urination, urgency and frequency. She is sexually active. Urine cultures show catalase-positive, gram-positive cocci. The organism responsible for this patient's symptoms is most likely to be:

Explanation

The correct answer is Novobiocin resistant. Novobiocin is an antibiotic that is used to differentiate between coagulase-negative staphylococci (which are resistant to novobiocin) and Staphylococcus saprophyticus (which is sensitive to novobiocin). In this case, the patient's symptoms of burning urination, urgency, and frequency, along with the presence of catalase-positive, gram-positive cocci in the urine cultures, suggest a urinary tract infection. Staphylococcus saprophyticus is a common cause of urinary tract infections in young sexually active females, and it is typically resistant to novobiocin. Therefore, the most likely organism responsible for this patient's symptoms is novobiocin resistant.

Submit
8. An 18-year-old college freshman is brought to the emergency room with a high fever, confusion and headaches. Physical examination reveals nuchal rigidity and a purpuric rash on his lower extremities. This infection could have been prevented by a vaccine containing:

Explanation

The correct answer is capsular polysaccharide. This is because the patient's symptoms, including fever, confusion, headaches, nuchal rigidity, and purpuric rash, are consistent with meningococcal meningitis, which is caused by the bacterium Neisseria meningitidis. The capsule of this bacterium is composed of polysaccharides, and vaccines containing capsular polysaccharide have been shown to be effective in preventing meningococcal infections.

Submit
9. A 58-year-old male presents to the ER with fever, headache and vomiting. His past medical history is significant for a recent kidney transplantation. Lumbar puncture reveals CSF pleocytosis and a normal CSF glucose concentration. Gram positive rods with tumbling motility at room temperature are seen on CSF microscopy. How was this patient's infection most likely transmitted?

Explanation

This patient's infection is most likely transmitted through contaminated food. The presence of gram positive rods with tumbling motility at room temperature in the CSF indicates infection with Listeria monocytogenes, which is commonly transmitted through the ingestion of contaminated food, especially unpasteurized dairy products, deli meats, and soft cheeses. This mode of transmission is consistent with the patient's symptoms of fever, headache, and vomiting.

Submit
10. A 23-year-old Caucasian female is brought to the ER with fever, vomiting, diarrhea and muscle pain. Her blood pressure is 90/50 mm Hg and pulse is 120/min. Physical examination reveals erythroderma, and pelvic exam reveals a tampon in the vagina. The activation of which of the following cells is primarily responsible for this patient's condition?

Explanation

The patient presents with symptoms of fever, vomiting, diarrhea, muscle pain, low blood pressure, and a tampon in the vagina. These findings suggest toxic shock syndrome (TSS), which is caused by an immune response to toxins produced by Staphylococcus aureus or Streptococcus pyogenes. Macrophages and T lymphocytes play a crucial role in the immune response against these bacteria. Macrophages phagocytose the bacteria and present antigens to T lymphocytes, which then activate and release cytokines, leading to systemic symptoms such as fever and hypotension. Therefore, the activation of macrophages and T lymphocytes is primarily responsible for this patient's condition.

Submit
11. A 22-year-old female presents with severe pain and swelling in her right knee, left elbow, and left wrist. Joint aspiration of the knee reveals an opaque exudate with high neutrophil content and intracellular organisms. This patient's symptoms could have most likely been prevented by:

Explanation

Barrier contraception refers to the use of physical barriers such as condoms or diaphragms to prevent the transmission of sexually transmitted infections (STIs) and unwanted pregnancies. The patient in this scenario presents with symptoms suggestive of a septic arthritis, which is typically caused by an infection. The presence of intracellular organisms in the joint aspiration suggests a potential sexually transmitted infection. Therefore, practicing barrier contraception would have reduced the risk of acquiring an STI, which could have prevented the symptoms of joint infection in this patient.

Submit
12. A developing country includes infant vaccination with the Haemophi/us inffuenzaetype b conjugate vaccine into its routine immunization schedule. If effectively implemented, this change in the vaccination schedule would most likely affect the epidemiology of:

Explanation

The introduction of the Haemophilus influenzae type b (Hib) conjugate vaccine into the routine immunization schedule of a developing country would most likely affect the epidemiology of meningitis. This is because Hib is one of the leading causes of bacterial meningitis, especially in children under the age of 5. By vaccinating infants against Hib, the country would significantly reduce the number of Hib infections and consequently decrease the incidence of meningitis caused by this bacterium.

Submit
13. Bacteria isolated from the lung tissue of a 32-year-old Caucasian male fail to decolorize with hydrochloric acid and alcohol after staining carbolfuchsin. Which of the following cell wall components is most likely responsible for this staining phenomenon?

Explanation

Mycolic acid is a component found in the cell wall of acid-fast bacteria, such as Mycobacterium tuberculosis. It is responsible for the staining phenomenon observed in acid-fast staining methods, such as the Ziehl-Neelsen stain, where bacteria retain the carbolfuchsin stain even after decolorization with acid and alcohol. This characteristic staining property of mycolic acid is due to its high lipid content and resistance to decolorization. Therefore, the failure of the bacteria isolated from the lung tissue to decolorize with hydrochloric acid and alcohol after staining with carbolfuchsin suggests the presence of mycolic acid in their cell wall.

Submit
14. A 5-year-old boy is brought to the ER with breathing difficulty, dysphagia, drooling and fever. His temperature is 39.4C (103.0 F). White blood cell count is 23,000/cmm with many band forms. Laryngoscopy in the operating room shows a swollen and cherry-red epiglottis. This patient most likely:

Explanation

The correct answer is "Missed vaccination" because the patient's symptoms are consistent with acute epiglottitis, which is most commonly caused by Haemophilus influenzae type B (Hib) infection. Hib vaccination is part of routine childhood immunization and has greatly reduced the incidence of epiglottitis. Therefore, a missed vaccination would increase the risk of developing this condition. The symptoms of breathing difficulty, dysphagia, drooling, and fever, along with the physical finding of a swollen and cherry-red epiglottis, support this diagnosis.

Submit
15. A 78-year-old nursing home resident develops high fevers, chest pain and cough productive of rusty sputum. Sputum microscopy reveals numerous lancet-shaped Gram positive cocci in pairs. The patient dies despite aggressive hydration and antibiotic treatment. A vaccine containing which of the following might have prevented this outcome?

Explanation

Capsular polysaccharide is the correct answer because the patient's symptoms and sputum microscopy findings are consistent with pneumococcal pneumonia, which is caused by Streptococcus pneumoniae. The capsule of S. pneumoniae is composed of polysaccharides, and it helps the bacteria evade the immune system. Vaccines containing capsular polysaccharides from different strains of S. pneumoniae have been developed and have been shown to be effective in preventing pneumococcal infections, including pneumonia. Therefore, a vaccine containing capsular polysaccharide might have prevented the outcome of the patient's death.

Submit
16. A sample of contaminated moist soil is heated to 1 oo• C for 15 minutes. Which of the following bacteria is most likely to be recovered from the soil sample following heat exposure?

Explanation

Bacillus anthracis is the most likely bacteria to be recovered from the soil sample following heat exposure because it is a spore-forming bacteria. Spore-forming bacteria have the ability to form dormant spores that can withstand extreme conditions, including high temperatures. Heating the soil to 100°C for 15 minutes would kill most bacteria, but spores can survive and later germinate when conditions become favorable again. Bacillus anthracis is known to form highly resistant spores, making it more likely to survive the heat exposure compared to the other bacteria listed.

Submit
17. A 55-year-old woman had her rheumatic heart valve replaced with a pros- thetic valve. Six blood cultures became positive after three days of incuba- tion. An optochin-resistant, catalase-negative gram-positive coccus that was alpha-hemolytic was isolated. What was the most likely causal agent?

Explanation

The most likely causal agent in this case is Streptococcus viridans. This is indicated by the fact that the isolated organism is a gram-positive coccus that is alpha-hemolytic, optochin-resistant, and catalase-negative. Streptococcus viridans is known to cause infective endocarditis in patients with prosthetic heart valves, making it the most likely explanation in this scenario.

Submit
18. Genital examination of a 31-year-old female reveals yellow discharge from the cervical os that demonstrates abundant neutrophils on light microscopy. The patient will most likely experience which of the following as a sequela of this disease?

Explanation

The presence of yellow discharge from the cervical os along with abundant neutrophils suggests the presence of an infection. Infections of the female reproductive system, such as pelvic inflammatory disease (PID), can lead to scarring and damage to the reproductive organs. This scarring can result in blockage or damage to the fallopian tubes, making it difficult for the egg to travel from the ovary to the uterus, leading to infertility. Therefore, the most likely sequela of this disease is infertility.

Submit
19. A 24-year-old female presents to your office with burning during urination, urine clouding and urinary frequency. She denies fever, chills and flank pain. She had a similar episode before that was treated with antibiotics. She has no other medical problems and does not use tobacco, alcohol or drugs. Her vital signs are stable. Physical examination shows suprapubic tenderness. Which of the following bacteria is most likely to be isolated from this patient's urine?

Explanation

Escherichia coli is the most likely bacteria to be isolated from this patient's urine because it is the most common cause of urinary tract infections (UTIs), especially in young, sexually active females. The patient's symptoms of burning during urination, urine clouding, and urinary frequency are consistent with a UTI. Additionally, the absence of fever, chills, and flank pain suggests that the infection has not yet spread to the kidneys. E. coli is a gram-negative bacterium that is commonly found in the gastrointestinal tract and can ascend into the urinary tract, causing infection.

Submit
20. A 3-year-old male experiences several episodes of otitis media. Middle ear exudate cultures grow H. inffuenzae although the patient's immunizations are up-to-date. Which of the following explains H. inffuenzae infection in this patient despite immunizations?

Explanation

The correct answer is that the strains responsible for this patient's disease do not produce a capsule. The capsule of H. influenzae is an important virulence factor that allows the bacteria to evade the immune system and cause infection. Without a capsule, the bacteria are more susceptible to the immune response and are less likely to cause disease. Therefore, even though the patient's immunizations are up-to-date, they are still susceptible to infection with non-capsulated strains of H. influenzae.

Submit
21. A 45-year-old female executive goes to a cosmetic surgeon with the com- plaint of frown lines on her forehead which she feels are negatively affecting her appearance. Rather than undergoing surgery, she opts to try injection of Botox. What is the mechanism of action of this toxin?

Explanation

Botox works by blocking the release of acetylcholine, a neurotransmitter responsible for muscle contraction. By inhibiting the release of acetylcholine, Botox temporarily paralyzes the muscles, preventing them from contracting and thus reducing the appearance of frown lines on the forehead. This mechanism of action is what allows Botox to be used as a cosmetic treatment for wrinkles and lines on the face.

Submit
22. Volunteer studies are used to determine the infective dose of Salmonella required to cause gastroenteritis. The curve obtained during the studies is shown on the diagram and is labeled as '2'. Which of the following organisms could be represented by line '1' on this diagram?

Explanation

The question asks which organism could be represented by line '1' on the diagram. Since the diagram is used to determine the infective dose of Salmonella, we can infer that line '1' represents an organism that causes gastroenteritis. Among the given options, Shigella nexneri is the only organism known to cause gastroenteritis, so it is the most likely organism represented by line '1' on the diagram.

Submit
23. A group of investigators is trying to develop an anti-gonococcal vaccine. Their attempts to use pilus components to induce long-lasting immunity would most likely fail because of:

Explanation

The reason why attempts to use pilus components to induce long-lasting immunity would most likely fail is due to antigenic variation. Gonococci, the bacteria responsible for gonorrhea, are known to have high levels of genetic variability, leading to frequent changes in their surface antigens. This antigenic variation allows the bacteria to evade the immune system and makes it difficult for the immune system to recognize and target them effectively. Therefore, using pilus components, which are part of the bacterial surface, as a vaccine target would not be effective in inducing long-lasting immunity.

Submit
24. A 39 year-old paraplegic man with an indwelling bladder catheter presents to the emergency room complaining of twenty-four hours of rigors, nausea, and vomiting. His temperature is 38.9° C (102.0° F). Urinalysis shows 3+ leukocyte esterase and numerous white blood cells. Urine and blood cultures grow non-lactose fermenting gram-negative bacilli. Which of the following pathogens is the most likely culprit?

Explanation

The most likely culprit in this case is Pseudomonas aeruginosa. This is because the patient's symptoms of rigors, nausea, and vomiting, along with a high temperature, are consistent with a urinary tract infection. The presence of leukocyte esterase and numerous white blood cells in the urinalysis suggests an inflammatory response in the urinary tract. Pseudomonas aeruginosa is a common cause of urinary tract infections in patients with indwelling bladder catheters, especially in those with underlying conditions such as paralysis. Additionally, the growth of non-lactose fermenting gram-negative bacilli in both urine and blood cultures further supports Pseudomonas aeruginosa as the likely pathogen.

Submit
25. A 54-year-old male presents to the ER with a sore on his right shoulder. Physical examination demonstrates an ulcer with central black eschar surrounded by edema. Exudate microscopy reveals chains of large Gram-positive rods. Which of the following would be most helpful in making the diagnosis?

Explanation

Occupation would be most helpful in making the diagnosis because the presentation of an ulcer with central black eschar surrounded by edema and the finding of chains of large Gram-positive rods on exudate microscopy suggests a diagnosis of cutaneous anthrax. Cutaneous anthrax is most commonly seen in individuals who work with animals or animal products, such as farmers, veterinarians, or butchers. Therefore, knowing the patient's occupation would provide important information to support the diagnosis of cutaneous anthrax.

Submit
26. A 67-year-old male is hospitalized with low-grade fevers, fatigue and a diastolic murmur at the left sternal border. Blood cultures reveal Gram positive cocci that are catalase-negative and able to grow in 6.5% saline. This patient's medical history is most likely to reveal which of the following procedures in the past month?

Explanation

The correct answer is cystoscopy. This is because the patient presents with low-grade fevers, fatigue, and a diastolic murmur, which are indicative of infective endocarditis. The presence of Gram positive cocci that are catalase-negative and able to grow in 6.5% saline suggests the causative organism is Streptococcus viridans, which is commonly associated with dental procedures. However, in this case, the patient's medical history is most likely to reveal a cystoscopy, as this procedure carries a risk of introducing bacteria into the bloodstream and causing infective endocarditis.

Submit
27. A 29-year-old female presents to clinic with malaise and an extremely painful rash in her genital region. She has never had such symptoms before. Tzanck smear of the genital lesion is positive for multinucleated giant cells. A drug that can cause rapid resolution of this patient's symptoms would most likely:

Explanation

The correct answer is "Incorporate into newly replicated viral DNA." This is because the presence of multinucleated giant cells on the Tzanck smear suggests a viral infection, such as herpes simplex virus. Drugs that incorporate into newly replicated viral DNA, such as nucleoside analogues like acyclovir, can inhibit viral DNA synthesis and prevent further replication of the virus. This leads to rapid resolution of symptoms.

Submit
28. A 7-year-old Caucasian boy is brought to your office with blisters on his face. Some of the blisters have broken and are covered with golden yellow crusts. Exudate microscopy reveals Gram-positive cocci in chains. Which of the following would be a component of the clinical syndrome that may follow such an infection?

Explanation

The presence of blisters on the boy's face, along with the golden yellow crusts and the identification of Gram-positive cocci in chains in the exudate microscopy, suggests that the boy has impetigo, which is a bacterial skin infection commonly caused by Staphylococcus aureus or Streptococcus pyogenes. In severe cases, such as with a systemic infection, complications can occur. One such complication is post-streptococcal glomerulonephritis, which can present with symptoms like face puffiness and dark urine.

Submit
29. A 24-year-old male presents to your ER with severe nausea and vomiting two hours after consuming a mayonnaise-containing salad at an outdoor picnic. His condition improved significantly in several hours. You inform the patient that his symptoms were caused by which of the following?

Explanation

The patient's symptoms of severe nausea and vomiting occurring two hours after consuming the salad suggest that the cause of his symptoms was an exotoxin formed prior to ingestion. Exotoxins are toxins released by bacteria that can cause symptoms even before the bacteria themselves are present in the body. In this case, the mayonnaise-containing salad likely contained bacteria that produced exotoxins, which caused the patient's symptoms.

Submit
30. Clustered Gram positive bacteria are isolated from the tonsillar exudates of a 6-year-old boy. On microscopic examination, the bacteria have polar granules that stain deeply with aniline dyes. The pathogenicity of these organisms is the result of:

Explanation

The presence of polar granules that stain deeply with aniline dyes suggests that the bacteria in question are likely Corynebacterium diphtheriae, the causative agent of diphtheria. Diphtheria toxin, produced by C. diphtheriae, is responsible for the pathogenicity of these organisms. The toxin inhibits protein synthesis in host cells, leading to cell death and tissue damage. Therefore, the impairment of protein synthesis is the correct answer explaining the pathogenicity of these bacteria.

Submit
31. Health authorities are investigating an outbreak of respiratory infections among a group of military recruits. Fifteen recruits have reported persistent cough, low-grade fever and malaise. Each patient's chest X-ray shows pulmonary infiltrates more severe in appearance than would be expected from the patient's clinical status. Sputum cultures require a complex acellular medium enriched with cholesterol in order to grow. Which of the following organisms is the most likely cause of the outbreak?

Explanation

The most likely cause of the outbreak is Mycoplasma pneumoniae. This is suggested by the symptoms reported by the recruits, such as persistent cough, low-grade fever, and malaise, which are consistent with a respiratory infection. The chest X-rays showing pulmonary infiltrates that are more severe than expected also align with Mycoplasma pneumoniae, as it is known to cause atypical pneumonia with extensive lung involvement. Additionally, the fact that sputum cultures require a complex acellular medium enriched with cholesterol to grow further supports Mycoplasma pneumoniae as the likely organism, as it is a fastidious bacterium that requires specific growth conditions.

Submit
32. A 7-day-old infant presents to the emergency department with a fever, poor feeding, and a bulging fontanelle. During her physical examination, she begins to convulse. A Gram stain of the CSF reveals gram-positive rods. Which of the following organisms is the most likely causal agent?

Explanation

Listeria monocytogenes is the most likely causal agent in this case because it is a gram-positive rod and is known to cause meningitis in neonates. The presentation of fever, poor feeding, bulging fontanelle, and convulsions is consistent with meningitis. Listeria monocytogenes is a common cause of bacterial meningitis in infants less than 1 month old, especially in premature infants. It can be transmitted to the infant through the placenta or during delivery. Prompt treatment with antibiotics is essential to prevent serious complications and mortality.

Submit
33. A 15-day-old boy presents with conjunctivitis. Iodine staining bodies are seen in conjunctival scrapings. The most likely infectious form is a(n)

Explanation

The presence of iodine staining bodies in conjunctival scrapings suggests the presence of elementary bodies. Elementary bodies are the infectious form of certain bacteria, such as Chlamydia trachomatis, which causes conjunctivitis. These small, round bodies are resistant to environmental conditions and can survive outside of host cells. They are responsible for initiating infection and are typically found in the initial stages of the disease. Reticulate bodies, endospores, exotoxins, and vegetative cells are not typically associated with conjunctivitis caused by Chlamydia trachomatis.

Submit
34. A 34-year-old woman comes to the physician because of itching around her vulva and vaginal discharge. She has no other complaints. Her other medical problems include chronic allergic rhinitis, for which she takes intranasal corticosteroids. She had an episode of acute bacterial sinusitis 2 week ago and received the appropriate therapy. Pelvic examination shows an erythematous vulva and a thick, adherent, "cottage cheeselike" vaginal discharge. The rest of the physical examination is within normal limits. Microscopic examination of the vaginal discharge reveals budding cells. Which of the following is the most likely underlying cause of her current condition?

Explanation

The presence of a thick, adherent "cottage cheeselike" vaginal discharge, along with microscopic examination revealing budding cells, suggests a yeast infection. Yeast infections are commonly caused by an overgrowth of Candida species, which are a type of fungus. The normal vaginal flora includes a balance of bacteria, including Gram-positive bacteria, which help maintain a healthy vaginal environment and prevent overgrowth of yeast. Therefore, a decreased number of Gram-positive bacteria in the vagina can disrupt this balance and contribute to the development of a yeast infection.

Submit
35. A 32-year-old Caucasian male develops profuse watery diarrhea with abdominal cramps, nausea and vomiting during a trip to Mexico. The diarrhea resolved in a couple of days without antibiotic treatment. This patient's symptoms are most likely related to:

Explanation

The patient's symptoms of profuse watery diarrhea, abdominal cramps, nausea, and vomiting are consistent with cholera. Cholera is caused by the bacterium Vibrio cholerae, which produces a toxin called cholera toxin. This toxin causes the intestines to secrete large amounts of water, leading to the characteristic watery diarrhea. The fact that the diarrhea resolved without antibiotic treatment also supports the diagnosis of cholera, as antibiotics are not typically needed for treatment.

Submit
36. An outbreak of water-borne gastroenteritis with a high mortality is reported in some Asian countries. It is caused by oxidase-positive, gram-negative rods that grow well on highly alkaline media. Which of the following patients would most likely require the smallest infective dose to initiate the infection?

Explanation

Patients on omeprazole treatment would most likely require the smallest infective dose to initiate the infection. Omeprazole is a proton pump inhibitor that reduces the production of gastric acid in the stomach. Gastric acid plays a crucial role in killing ingested bacteria, so when its production is reduced, the stomach becomes less acidic and more susceptible to bacterial infection. Therefore, patients on omeprazole treatment have a higher risk of developing infections, including water-borne gastroenteritis caused by oxidase-positive, gram-negative rods that grow well on highly alkaline media.

Submit
37. A 5-month-old Hispanic boy is brought to the ER with complaints of poor feeding, weakness and complete loss of extremity muscle tone. All of his vaccinations are up to date and there is no significant past medical history. He receives formula as his sole source of nutrition with the exception of occasional fruit juice and honey. He has also received vitamin D supplementation. Which of the following tests is most likely to establish the diagnosis in this patient?

Explanation

The most likely diagnosis in this patient is botulism, a condition caused by the ingestion of the bacterial toxin produced by Clostridium botulinum. The symptoms of poor feeding, weakness, and loss of muscle tone are consistent with botulism. Stool for bacterial toxins is the most appropriate test to establish the diagnosis as it can detect the presence of the toxin in the patient's stool. Blood tests for liver enzymes or viral titers would not be helpful in this case. Urine tests for glucose and ketones or amino acids would also not provide relevant information for the diagnosis of botulism.

Submit
38. A 12-year-old boy is brought to the physician with fever, chills, and a rash that appeared this morning. Two days earlier, the child started complaining of a sore throat. His temperature is 38.3° C (101° F}. Examination shows a diffuse ery1hematous rash on his chest and abdomen that blanches with pressure, along with numerous 1- to 2-mm papules. The throat is ery1hematous with gray-white tonsillar exudates and the tongue is bright red. Which of the following is the most serious complication that can develop from this disease?

Explanation

The most serious complication that can develop from this disease is rheumatic fever. Rheumatic fever is a systemic inflammatory disease that can occur as a complication of untreated or inadequately treated streptococcal pharyngitis. It typically occurs 2-3 weeks after the initial infection and can affect the heart, joints, skin, and brain. It can lead to permanent damage to the heart valves, causing rheumatic heart disease, which can be life-threatening.

Submit
39. Autopsy of a 14-year-old male who died of septic shock shows extensive hyperemia and infarcts of his internal organs. Purpuric cutaneous lesions and hemorrhagic destruction of both adrenals are also present. The organism most likely responsible for this patient's death more commonly causes which of the following?

Explanation

The autopsy findings of hyperemia and infarcts of internal organs, purpuric cutaneous lesions, and hemorrhagic destruction of both adrenals suggest disseminated intravascular coagulation (DIC). DIC is often caused by meningococcal meningitis, which can lead to septic shock and multiorgan failure. Therefore, the organism most likely responsible for this patient's death, meningococcus, commonly causes meningitis.

Submit
40. A group of investigators has developed monoclonal antibodies to the meningococcal pilus. They believe that these antibodies may decrease the risk of infection by meningococci. If effective, these antibodies would directly interfere with which of the following processes?

Explanation

Monoclonal antibodies to the meningococcal pilus would directly interfere with pharyngeal colonization. This means that the antibodies would prevent the bacteria from colonizing or establishing an infection in the pharynx, which is the area at the back of the throat. By targeting and inhibiting this colonization process, the antibodies would decrease the risk of infection by meningococci.

Submit
41. A county hospital experiences an outbreak of methicillin-resistant Staphylococcus aureus (MRSA) wound infections. You suspect that many staff members are carriers of the bacteria. Which of the following would you expect to be the most commonly MRSA-populated site in staff members?

Explanation

The most commonly MRSA-populated site in staff members would be the anterior nares. This is because the anterior nares, or the nostrils, are a common colonization site for Staphylococcus aureus, including the methicillin-resistant strains. The bacteria can easily colonize the anterior nares and be transmitted through direct contact or respiratory droplets. Therefore, it is likely that staff members who are carriers of MRSA would have the bacteria present in their anterior nares.

Submit
42. A 46-year-old female presents to the emergency room with fever, chest pain and hemoptysis. She came to visit her family from Pakistan where she works in a goat wool processing center. Chest x-ray shows widened mediastinum. Sputum and blood cultures reveal large gram-positive rods that form 'medusa head' colonies on standard media. The bacteria isolated from this patient most likely produce:

Explanation

The correct answer is "Antiphagocytic D-glutamate capsule". This is because the patient's symptoms, including fever, chest pain, and hemoptysis, along with the widened mediastinum on chest x-ray, are consistent with a diagnosis of anthrax. The presence of large gram-positive rods that form "medusa head" colonies on standard media further supports this diagnosis. Anthrax is caused by the bacteria Bacillus anthracis, which is known to produce an antiphagocytic D-glutamate capsule. This capsule helps the bacteria evade the immune system and contributes to its pathogenicity.

Submit
43. A 10-year-old Caucasian male is brought to the ER with a several day history of high fevers and chills. He also complains of dull pain just above the left knee. There is no joint effusion. X-ray films show soft tissue swelling, bone destruction, and periosteal reaction over the lower end of femur. Which of the following organisms is most likely responsible for this patient's symptoms?

Explanation

The patient's symptoms, including high fevers, chills, and pain above the left knee, along with the findings on X-ray films of soft tissue swelling, bone destruction, and periosteal reaction, are indicative of osteomyelitis. Staphylococcus aureus is the most common causative organism for acute hematogenous osteomyelitis, especially in children. Therefore, Staphylococcus aureus is the most likely responsible organism for this patient's symptoms.

Submit
44. What is the structure that is found in gram-negative but not in gram- positive bacteria?

Explanation

Gram-negative bacteria have an outer membrane, which is an additional layer outside the cell wall. This outer membrane is absent in gram-positive bacteria. The outer membrane in gram-negative bacteria provides an extra barrier of protection and helps to prevent the entry of certain substances, such as antibiotics, into the cell. It also contains specific proteins called porins, which allow the passage of certain molecules. This structural difference between gram-negative and gram-positive bacteria is important in understanding their different responses to antibiotics and other substances.

Submit
45. A 65-year-old man develops pneumonia. The organisms isolated from the sputum are gram-positive cocci that are alpha hemolytic on blood agar and sensitive to optochin. Which structure of the causal agent provides protec- tion against phagocytosis?

Explanation

The correct answer is Capsule. The presence of a capsule provides protection against phagocytosis by preventing the immune system cells from engulfing the bacteria. The capsule acts as a barrier, making it difficult for phagocytes to recognize and engulf the bacteria. This allows the bacteria to evade the immune system and survive in the host, leading to the development of pneumonia.

Submit
46. A 38-year-old man who recently visited India on business presents to the emergency department with profuse watery diarrhea flecked with mucus, and severe dehydration. Which of the following correctly describes the causal agent?

Explanation

The correct answer describes the causal agent as a Gram-negative curved rod that produces a toxin which increases cAMP levels. This is consistent with the presentation of profuse watery diarrhea flecked with mucus, which is characteristic of cholera caused by Vibrio cholerae. The toxin produced by Vibrio cholerae, called cholera toxin, increases cAMP levels in the intestinal epithelial cells, leading to the secretion of large amounts of fluid into the intestinal lumen and subsequent severe dehydration.

Submit
47. A 70-year-old man is hospitalized for an infection and treated with clindamy- cin. The patient improves and returns to his nursing home. Two weeks later he is rushed to the emergency room with fever and loose, mucoid green stools. The diarrhea is voluminous, and he is having severe abdominal pain. Sigmoidoscopy of his colon reveals yellow-white plaques. What is the single most likely event/factor that contributed to this patient's current illness?

Explanation

The administration of antibiotics is the single most likely event/factor that contributed to this patient's current illness. This is because the patient was treated with clindamycin, which is known to disrupt the normal flora of the gut and can lead to an overgrowth of Clostridium difficile. This overgrowth can cause a severe infection known as Clostridium difficile-associated diarrhea (CDAD), which is characterized by fever, loose mucoid green stools, voluminous diarrhea, and severe abdominal pain. The yellow-white plaques seen on sigmoidoscopy are consistent with pseudomembranous colitis, a complication of CDAD.

Submit
48. A 23-year-old Caucasian female presents to clinic with lower abdominal pain and scant bloody vaginal discharge. She has been sexually active with multiple partners and uses condoms on occasion. She has been treated for "genital infections" in the past but denies any history of pregnancy. Her blood pressure is 11 2/70 mm Hg while supine and 96/60 mm Hg while standing. A pregnancy test is positive. Which of the following microorganisms is most likely responsible for this patient's current condition?

Explanation

The correct answer is Neisseria gonorrhoeae. This patient's presentation of lower abdominal pain, scant bloody vaginal discharge, history of multiple sexual partners, and positive pregnancy test is consistent with pelvic inflammatory disease (PID), which is most commonly caused by Neisseria gonorrhoeae. Other symptoms such as a history of "genital infections" and previous treatment further support this diagnosis.

Submit
49. A 35-year-old man presents to your office complaining of a hard mass under his jaw. He reports that the mass developed three months ago, shortly after he had a tooth extracted. The mass has since grown in size, and has recently begun to drain yellowish pus through the overlying skin. Which of the following is the most likely cause of this patient's condition?

Explanation

The most likely cause of the patient's condition is Actinomyces israelii. Actinomyces is a gram-positive bacteria that can cause chronic infections in the head and neck region. The patient's history of tooth extraction and the development of a hard mass under his jaw, which has grown in size and is now draining pus, are consistent with actinomycosis. Actinomyces israelii is known to cause this type of infection, which often presents as a firm, non-tender mass with sinus tract formation and purulent drainage.

Submit
50. Epidemiological analyses show a significant increase in staphylococcal infections (bacteremia and sepsis) in several large city hospitals over the last 20 years. This increase in the infection incidence most closely correlates with the increased use of:

Explanation

The correct answer is "lntravascular devices" because staphylococcal infections, such as bacteremia and sepsis, are commonly associated with the use of these devices. Intravascular devices, such as central venous catheters or indwelling urinary catheters, provide a direct pathway for bacteria to enter the bloodstream, leading to infection. The increased use of these devices in hospitals over the last 20 years is likely contributing to the significant increase in staphylococcal infections observed in the epidemiological analyses.

Submit
51. A group of researchers is attempting to determine the asymptomatic carrier rates of certain bacterial pathogens in various age groups. They sample and culture pharyngeal exudates from individuals of different ages. Healthy volunteers who have no significant medical history, take no medication, and have been free of upper respiratory tract illnesses for the past 6 weeks are recruited. A pharyngeal exudate sample from a woman in the 20-25-year-old group is placed on a selective medium containing vancomycin, colistin, nystatin and trimeihoprim. This medium favors growth of which of the following bacteria?

Explanation

The selective medium containing vancomycin, colistin, nystatin, and trimethoprim is specifically designed to favor the growth of Neisseria gonorrhoeae. This bacterium is the causative agent of gonorrhea, a sexually transmitted infection that commonly affects the pharynx. By using this selective medium, the researchers can isolate and identify asymptomatic carriers of N. gonorrhoeae in the 20-25-year-old age group. The other bacteria listed are not specifically targeted by this selective medium and are less likely to grow under these conditions.

Submit
52. Sixteen residents in a retirement home have fever, malaise, and anorexia. These residents have taken their meals prepared by the same kitchen. Blood cultures from 11 of these residents grow Salmonella enterica subsp. typhi. The primary reservoir of this organism is

Explanation

The correct answer is people because the question states that the residents in the retirement home have taken their meals prepared by the same kitchen. Since blood cultures from 11 of these residents grow Salmonella enterica subsp. typhi, it suggests that the primary reservoir of this organism is people. This means that the residents themselves are the source of the Salmonella infection, possibly due to poor hygiene or contaminated food handling practices.

Submit
53. A tourist who recently returned from a trip to Peru goes to her physician complaining of persistent high fever, malaise, and constipation that per- sisted over a week. She recalls that the fever began slowly and climbed to 41.0 C (105.8 F). A physical exam reveals an enlarged spleen and tender abdomen with rose-colored spots. Laboratory isolation of a bacterium that produces H2S and is motile is revealed. Which organism is the most likely cause of her condition?

Explanation

The correct answer is Salmonella enterica subsp. typhi. This is the most likely cause of the tourist's condition based on the symptoms described. The persistent high fever, malaise, constipation, enlarged spleen, and rose-colored spots are all consistent with typhoid fever, which is caused by Salmonella enterica subsp. typhi. The isolation of a bacterium that produces H2S and is motile further supports this diagnosis, as these are characteristic features of Salmonella enterica subsp. typhi.

Submit
54. A 60-year-old woman is hospitalized following a stroke and develops a high-grade fever with chills. She is catheterized due to urinary incontinence and receives cephalosporin for treatment of pneumonia. Blood cultures and Gram stain are performed by the laboratory. The organisms isolated are gram-positive cocci that are catalase-negative and capable of growth in 6.5% sodium chloride. Which of the following is the most likely causal agent?

Explanation

The most likely causal agent in this case is Enterococcus faecalis. This is because the question mentions that the organisms isolated are gram-positive cocci that are catalase-negative and capable of growth in 6.5% sodium chloride. Enterococcus faecalis fits this description as it is a gram-positive cocci bacterium that is catalase-negative and can grow in high salt concentrations. Additionally, Enterococcus faecalis is known to cause urinary tract infections and is commonly associated with hospital-acquired infections.

Submit
55. A 15-year-old girl develops a sore throat, fever, and earache of approximately one week duration. Upon examination by her physician, an erythematous rash is noted covering most of her body and her tongue appears bright red. Which of the following is the description of the causal agent?

Explanation

The description of the causal agent is "Gram-positive coccus, beta hemolytic, catalase negative." This means that the organism responsible for the girl's symptoms is a type of bacteria that appears purple when stained with the Gram stain, is able to completely break down red blood cells (beta hemolysis), and does not produce the enzyme catalase. This information helps to narrow down the possible bacterial causes of her symptoms and guide treatment decisions.

Submit
56. A 62-year-old Caucasian male who recently underwent a mitral valve replacement is having low-grade fevers. He also complains of dyspnea and malaise. Repeated blood cultures grow Gram-positive cocci in clusters that are catalase-positive and coagulase-negative. Which of the following is the best initial treatment for this patient?

Explanation

The patient in this scenario is presenting with low-grade fevers, dyspnea, and malaise after undergoing a mitral valve replacement. The repeated blood cultures have grown Gram-positive cocci in clusters that are catalase-positive and coagulase-negative. These characteristics are consistent with a staphylococcal infection, specifically coagulase-negative staphylococci. Vancomycin is the best initial treatment choice for this patient because it has excellent activity against Gram-positive bacteria, including methicillin-resistant Staphylococcus aureus (MRSA), which is a concern in patients who have undergone surgery.

Submit
57. A 52-year-old male is having low-grade fevers after recent replacement of his aortic valve. Repeated blood cultures grow gram-positive cocci. Which of the following characteristics is most consistent with Staphylococcus epidermidis as the cause of this patient's symptoms?

Explanation

Staphylococcus epidermidis is a coagulase-negative bacterium, meaning it does not produce the enzyme coagulase. This characteristic is consistent with the patient's symptoms of low-grade fevers after aortic valve replacement, as S. epidermidis is a common cause of postoperative infections, particularly in prosthetic devices. Coagulase-negative bacteria are often associated with healthcare-associated infections and are known for their ability to form biofilms on medical devices.

Submit
58. A 46-year-old male who just returned from a mission trip to Latin America presents to your office complaining of fever, myalgias, dull abdominal pain, and a week-long history of watery diarrhea that has recently become bloody. Physical examination reveals a fever of 38.9° C (102.0° F), hepatosplenomegaly, and rose-colored spots at the periumbilical area. Which of the following is the most likely cause of this patient's symptoms?

Explanation

The patient's symptoms, including fever, myalgias, abdominal pain, and bloody diarrhea, along with the presence of hepatosplenomegaly and rose-colored spots at the periumbilical area, are consistent with a diagnosis of typhoid fever. Salmonella typhi is the causative agent of typhoid fever, which is commonly acquired through contaminated food or water in regions with poor sanitation. Other options such as Escherichia coli, Salmonella enteritidis, and Campylobacter fetus can also cause gastroenteritis, but they do not typically present with hepatosplenomegaly and rose-colored spots. Therefore, Salmonella typhi is the most likely cause of this patient's symptoms.

Submit
59. A 22-year-old male ingests a solution containing "C-labeled urea. He is then asked to blow into a tube, and the labeled carbon is detected in his breath samples. This test is most likely part of the evaluation for which of the following conditions?

Explanation

The test described in the question is called the urea breath test. It is used to diagnose Helicobacter pylori infection, which is a common cause of duodenal ulcers. When the labeled urea is ingested, it is broken down by the bacterium, releasing labeled carbon dioxide. This labeled carbon dioxide is then detected in the breath samples, confirming the presence of H. pylori infection and duodenal ulcer. Therefore, the test is most likely part of the evaluation for duodenal ulcer.

Submit
60. E coli strains isolated from a 4-year-old Caucasian female with bloody diarrhea produce a substance that inhibits protein synthesis in human cells. The substance shares many properties with the toxin produced by:

Explanation

The correct answer is Shigella dysenteriae because it is known to produce a toxin that inhibits protein synthesis in human cells, which is similar to the substance produced by the E. coli strains isolated from the 4-year-old with bloody diarrhea. Salmonella typhi, Pseudomonas aeruginosa, Proteus mirabilis, Vibrio cholerae, and Clostridium difficile do not produce a toxin with the same properties.

Submit
61. A 6-year-old male is brought to the pediatric emergency room with fever and sore throat. The parents tell you that the child has not received any immunizations. Physical exam reveals a grey pharyngeal exudate and Gram stain shows scant Gram positive organisms. Which of the following culture types would facilitate the growth of this organism?

Explanation

Cysteine-tellurite agar would facilitate the growth of the organism causing the symptoms in the 6-year-old male. Cysteine-tellurite agar is a selective medium used for the isolation and identification of Corynebacterium diphtheriae, the bacteria responsible for diphtheria. The grey pharyngeal exudate and the presence of scant Gram positive organisms on the Gram stain are consistent with a possible diphtheria infection. Cysteine-tellurite agar contains cysteine, which enhances the growth of C. diphtheriae, and tellurite, which inhibits the growth of other bacteria. Therefore, this medium would be the most appropriate for isolating and identifying the causative organism in this case.

Submit
62. A 44-year-old man is brought to the emergency department by his wife because of confusion that began a few hours ago. Earlier in the day, he complained of a severe headache and nausea. On physical examination, his temperature is 38.3 C (101 F) and nuchal rigidity is present. Lumbar puncture and cerebrospinal fluid (CSF) examination show: Gram stain of the CSF would most likely reveal which of the following?

Explanation

The presence of nuchal rigidity and the patient's symptoms of headache and nausea suggest that he may have meningitis. Gram stain of the cerebrospinal fluid (CSF) can provide important information about the causative organism. In this case, the most likely finding on Gram stain would be lancet-shaped, Gram-positive cocci in pairs. This is consistent with the most common cause of bacterial meningitis, which is Streptococcus pneumoniae.

Submit
63. The vaccine that provides protection against H. inffuenzae contains capsular polysaccharide of H. inffuenzae type b as well as diphtheria toxoid. The conjugated diphtheria toxoid to the polysaccharide in this vaccine:

Explanation

The conjugated diphtheria toxoid to the polysaccharide in this vaccine increases immunogenicity. This means that it enhances the ability of the vaccine to stimulate an immune response in the body. By combining the polysaccharide with the diphtheria toxoid, the vaccine is able to elicit a stronger immune response, leading to a more effective protection against H. influenzae.

Submit
64.  A 5-year-old child of an Eastern European immigrant family is brought to your pediatric clinic. The child is afebrile, but weak and exhausted from a week of paroxysmal coughing with inspiratory whoops, frequently associ- ated with vomiting. The parents profess religious objections to childhood vaccinations, but permit withdrawal of a blood sample, which reveals a lymphocytosis of 44,000/mm3. Production of lymphocytosis, insulin secre- tion, and histamine sensitization are all results of which attribute of this organism?

Explanation

Pertussis toxin is responsible for the production of lymphocytosis, insulin secretion, and histamine sensitization in this organism.

Submit
65. A 10-year-old child develops glomerulonephritis a week after he was treated for a sore throat. The causal agent is identified by serotyping of the

Explanation

The correct answer is M proteins. Glomerulonephritis is often caused by a group A Streptococcus infection, which can occur after a sore throat. The M proteins are a major virulence factor of group A Streptococcus and are responsible for the development of glomerulonephritis. These proteins help the bacteria evade the immune system and can also cross-react with host tissues, leading to an autoimmune response and damage to the kidneys. Therefore, the identification of M proteins through serotyping can help determine the causal agent in this case.

Submit
66. A 10-year-old girl with an incomplete vaccination history presents to her pediatrician with a fever of 38.6 C (101.5 F), sore throat, malaise, and difficulty breathing. Physical examination reveals cervical lymphadenopathy and a gray, leathery exudate in the rear of the oropharynx. The area bleeds profusely when disturbed with a tongue depressor. Which of the following correctly describes the causal agent?

Explanation

The correct answer is "Gram-positive aerobic rod; toxin that inhibits protein synthesis." This is because the description of the causal agent matches that of Corynebacterium diphtheriae, which is a Gram-positive aerobic rod. Corynebacterium diphtheriae produces diphtheria toxin, which inhibits protein synthesis in host cells, leading to the characteristic symptoms of diphtheria such as sore throat, malaise, and difficulty breathing. The presence of cervical lymphadenopathy and a gray, leathery exudate in the oropharynx further supports this diagnosis.

Submit
67. An 8-year-old Caucasian boy is brought to your office with throat pain, fever and malaise. Physical examination reveals white exudates on his tonsils and swollen anterior cervical lymph nodes. If the boy returns in a month with fatigue, joint pain and chest pain and later in life he develops a heart murmur, which of the following would be the most likely explanation for his condition?

Explanation

The most likely explanation for the boy's condition is bacterial and human epitope homology. This means that the bacteria causing his initial symptoms have antigens that are similar to antigens found in his own body. This can lead to an autoimmune response, where the immune system mistakenly attacks the boy's own tissues, resulting in symptoms such as fatigue, joint pain, chest pain, and eventually a heart murmur. This condition is often seen in certain bacterial infections, such as streptococcal infections, which can lead to rheumatic fever.

Submit
68. A 45-year-old sailor comes to the physician with groin pain and swelling. He first noticed a sore on his penis but ignored it because it was not painful and disappeared after a few days. Three weeks later, he began to experience painful swelling in his inguinal region with inflammation of the overlying skin and the eventual formation of several draining ulcers. He also complains of mild fever and malaise that began around the same time as his groin symptoms. Cell scrapings from his lesions show cytoplasmic inclusion bodies. Which of the following is the most likely cause of this patient's condition?

Explanation

The patient's symptoms of painful swelling in the inguinal region with inflammation of the overlying skin and the eventual formation of several draining ulcers, along with the presence of cytoplasmic inclusion bodies in cell scrapings from his lesions, are consistent with lymphogranuloma venereum (LGV), which is caused by Chlamydia trachomatis. Haemophilus ducreyi causes chancroid, not the symptoms described. Klebsiella inguinale, also known as Donovanosis, presents with painless genital ulcers that do not resolve spontaneously. Herpes simplex virus causes painful vesicles that ulcerate and heal within a few weeks. Treponema pallidum causes syphilis, which presents with a painless chancre that heals spontaneously.

Submit
69. A 34-year-old male presents to your office with a painless penile ulcer several weeks after an episode of unprotected sexual intercourse. Which of the following is most useful in diagnosing Treponema pallidum infection in this patient?

Explanation

Ulcer exudate microscopy with darkfield illumination is the most useful in diagnosing Treponema pallidum infection in this patient. Darkfield microscopy allows for the visualization of the spirochetes, such as Treponema pallidum, which causes syphilis. This method is particularly effective in detecting the presence of the bacteria in the ulcer exudate, as it provides high contrast and enhances the visibility of the spirochetes. Other methods, such as Gram staining or blood cultures, may not be as effective in diagnosing syphilis specifically.

Submit
70. An 8-year-old male is brought to the ER with a two day history of fever, abdominal pain and diarrhea. Careful history taking reveals that the patient's pet puppy had diarrhea one week ago. Aside from the present episode, the patient has no significant past medical history and all of his vaccinations are up-to-date. His stool is negative for ova and parasites. Which of the following is the most likely cause of this patient's symptoms?

Explanation

The most likely cause of this patient's symptoms is Campylobacter. The patient's history of fever, abdominal pain, and diarrhea, along with the fact that the patient's pet puppy had diarrhea one week ago, suggests an infectious cause. Campylobacter is a common cause of bacterial gastroenteritis, often transmitted through contaminated food or water, and can cause symptoms such as fever, abdominal pain, and diarrhea. Other options such as Staphylococcus aureus, Shigella, Bacillus cereus, and Vibrio parahemolyticus are less likely based on the given information.

Submit
71. Non-pathogenic strains of Corynebacterium diphtheriae can acquire pathogenicity and thus the ability to cause severe pseudomembranous pharyngitis through which of the following mechanisms?

Explanation

Phage conversion is a mechanism by which non-pathogenic strains of Corynebacterium diphtheriae can acquire pathogenicity. This occurs when a bacteriophage infects the non-pathogenic strain and introduces the gene for the production of the diphtheria exotoxin. The exotoxin is a key virulence factor that causes severe pseudomembranous pharyngitis, a characteristic symptom of diphtheria. Therefore, phage conversion is the mechanism by which non-pathogenic strains can become pathogenic and cause disease.

Submit
72. A 34-year-old male is hospitalized with difficulty swallowing and blurred vision of sudden onset. He has never had symptoms like this before. Routine nerve stimulation studies show normal nerve conduction velocity but decreased compound muscle action potential. The patient should be carefully questioned about:

Explanation

The correct answer is home-canned food consumption. The patient's symptoms of difficulty swallowing and blurred vision of sudden onset suggest the possibility of botulism, which is caused by the ingestion of the botulinum toxin produced by Clostridium botulinum bacteria. Home-canned foods are a common source of botulism toxin, as the bacteria can grow in improperly canned foods. Therefore, it is important to question the patient about their consumption of home-canned foods to assess the likelihood of botulism as the cause of their symptoms.

Submit
73. A 32-year-old male presents to the emergency room with progressive difficulty walking over the last two days accompanied by tingling in his feet. Physical examination reveals decreased muscular strength in the bilateral lower extremities with absent deep tendon reflexes. His review of systems and past medical history are unremarkable except for a bout of diarrhea two weeks ago that resolved spontaneously. Which of the following agents is most likely associated with this patient's condition?

Explanation

The patient's symptoms of progressive difficulty walking, tingling in the feet, decreased muscular strength, and absent deep tendon reflexes are consistent with Guillain-Barré syndrome (GBS). GBS is an autoimmune condition that often occurs after an infection, and Campylobacter is one of the most common bacterial causes of GBS. The patient's history of diarrhea two weeks ago suggests an infection, and Campylobacter is a common cause of bacterial gastroenteritis. Therefore, Campylobacter is the most likely agent associated with the patient's condition.

Submit
74. Gram-positive bacteria are inoculated under the skin of experimental animals and then the infection is treated with antibiotics. Bacteria isolated from the injection site several days later assume a spherical configuration when placed in an isotonic solution and disintegrate rapidly when placed in a hypotonic solution. Which of the following antibiotic was most likely used in this experiment?

Explanation

The bacteria isolated from the injection site assume a spherical configuration when placed in an isotonic solution and disintegrate rapidly when placed in a hypotonic solution. This suggests that the bacteria have a thick peptidoglycan layer in their cell wall, which is a characteristic of Gram-positive bacteria. Cefuroxime is a type of antibiotic that targets the cell wall synthesis of bacteria, making it effective against Gram-positive bacteria. Therefore, cefuroxime is the most likely antibiotic used in this experiment.

Submit
75. Microscopic examination of a sputum sample from a 34-year-old male with fever and cough reveals Gram positive lancet-shaped cocci in pairs. These bacteria are likely to be:

Explanation

The bacteria in the sputum sample are likely to be bile soluble. This is because the presence of Gram positive lancet-shaped cocci in pairs, along with the fact that they are bile soluble, suggests that the bacteria are Streptococcus pneumoniae. Streptococcus pneumoniae is a common cause of pneumonia and is known to be bile soluble.

Submit
76. A 35-year-old male hospitalized with extensive trauma following a tractor accident complains of severe pain in his injured right leg. The leg swells rapidly, and radiographs reveal gas in the tissues. The microorganism most likely responsible for this patient's condition is also commonly associated with:

Explanation

The correct answer is "Transient watery diarrhea." The patient's symptoms of gas in the tissues and severe pain in the injured leg are consistent with gas gangrene, which is most commonly caused by Clostridium perfringens. This bacterium is also commonly associated with transient watery diarrhea, making it the most likely microorganism responsible for the patient's condition.

Submit
77. A toxic substance produced by C. perfringens induces massive hemolysis and tissue necrosis when injected into mice. The lethal effect observed in the experiment correlates with the substance's ability to split:

Explanation

The correct answer is phospholipids. Phospholipids are a major component of cell membranes and play a crucial role in maintaining the integrity and function of cells. When injected into mice, the toxic substance produced by C. perfringens causes massive hemolysis (breakdown of red blood cells) and tissue necrosis (death of cells and tissues). This suggests that the substance targets and disrupts the phospholipids in the cell membranes, leading to the observed lethal effects.

Submit
78. A homeless, malnourished chronic alcoholic presents with severe headache and dyspnea. Physical examination reveals a disheveled man with poor hygiene. His temperature is 41.0 C (105.8 F), blood pressure is 110/78 mm Hg, and pulse is 96/minute and regular. Auscultation of the chest reveals absence of breath sounds over the left middle lung fields. A chest x-ray con- firms left lobar pneumonia. Sputum stain reveals partially acid-fast bacilli with branching rods. Which of the following agents is the most likely cause?

Explanation

The patient's presentation with severe headache, dyspnea, and absence of breath sounds over the left middle lung fields suggests a pulmonary infection. The sputum stain revealing partially acid-fast bacilli with branching rods points towards Nocardia asteroides as the most likely cause. Nocardia asteroides is a gram-positive, partially acid-fast, branching filamentous bacterium that can cause pulmonary infections, especially in immunocompromised individuals. Mycobacterium avium-intracellulare, Mycobacterium kansasii, Mycobacterium leprae, and Mycobacterium tuberculosis are all mycobacteria but are less likely to be the cause in this case based on the clinical presentation and sputum stain results.

Submit
79. A 6-year-old girl had crashed on a toboggan ride and complained of pain in the perineal area. Exam showed only bruising of the area. Two days later, she develops fever, prostration, discoloration of the buttock, and blebs of the skin in the area. After admission to the hospital, she develops progres- sive involvement of the leg, thigh, and buttock with extension to the lower abdomen. She goes into shock and dies before surgery could be performed. At autopsy, a 1-inch piece of wood is found in the perineum, which had perforated the anus. The most likely causal agent

Explanation

The most likely causal agent in this case is a marked lecithinase producer. Lecithinase is an enzyme that breaks down lecithin, a phospholipid found in cell membranes. The presence of blebs and discoloration of the skin in the affected area suggests tissue damage, which could be caused by the action of lecithinase. Additionally, the progression of symptoms from the perineal area to the leg, thigh, and buttock, along with the development of shock, indicates a spreading infection. A marked lecithinase producer would be capable of causing such severe tissue damage and systemic infection.

Submit
80. A 70-year-old woman is brought to the emergency department by her spouse with complaints of shortness of breath and fever. Physical examina- tion revealed a fever of 39.4 C (103.0 F), hypotension, and a diastolic mur- mur. History revealed a cardiac valve replacement five years earlier. Three consecutive blood cultures taken during febrile periods revealed gram- positive cocci that were catalase-positive and coagulase-negative. Which of the following organisms is the most likely cause?

Explanation

Staphylococcus epidermidis is the most likely cause of the woman's symptoms. The patient's history of cardiac valve replacement suggests the possibility of infective endocarditis, and the presence of a diastolic murmur further supports this diagnosis. Staphylococcus epidermidis is a common cause of infective endocarditis in patients with prosthetic heart valves. The organism is catalase-positive and coagulase-negative, which matches the characteristics of the gram-positive cocci isolated from the patient's blood cultures.

Submit
81. The clinical laboratory reports the presence of 0157:H7 strains of E. coli in the bloody stools of six children ages 3-5 who attended a local petting zoo. These young children would be at an increased risk for developing

Explanation

The presence of 0157:H7 strains of E. coli in the bloody stools of the children indicates an infection with this particular strain of bacteria. Hemolytic uremic syndrome (HUS) is a serious complication that can occur after an E. coli infection, especially in young children. HUS is characterized by the destruction of red blood cells, low platelet count, and kidney failure. Therefore, these young children would be at an increased risk for developing hemolytic uremic syndrome.

Submit
82. Two days after eating a meal that included home-canned green beans, three people developed various degrees of visual problems, including double vision and difficulties focusing. Describe the Gram reaction of the organ- ism most likely to be isolated from the leftover beans and lab findings which would be used in its identification.

Explanation

The correct answer suggests that the organism most likely to be isolated from the leftover beans is a gram-positive bacillus that can grow on a low oxidation-reduction medium. This means that the organism is a type of bacteria that has a thick cell wall and stains purple in the Gram stain test. It is also able to grow in an environment with low oxygen levels. These characteristics can help in identifying the organism and determining its potential role in causing the visual problems experienced by the individuals who consumed the home-canned green beans.

Submit
83. Which genetic material is found in pathogenic Corynebacterium diphtheriae but not in nonpathogenic normal flora diphtheroids?

Explanation

Pathogenic Corynebacterium diphtheriae contains an integrated temperate phage, which is not found in nonpathogenic normal flora diphtheroids. This integrated temperate phage is a type of bacteriophage that has inserted its genetic material into the bacterial genome. This phage can remain dormant within the bacterial cell, but under certain conditions, it can become active and initiate the production of toxins that contribute to the pathogenicity of C. diphtheriae. Nonpathogenic diphtheroids, on the other hand, do not possess this integrated temperate phage.

Submit
84. How is a prophage created?

Explanation

A prophage is created through site-specific recombination of a temperate phage and bacterial DNA. This process involves the integration of the temperate phage's DNA into the bacterial DNA, resulting in the prophage. The temperate phage is able to exist in a dormant state within the bacterial cell, replicating along with the host DNA. This integration occurs at specific sites in the bacterial DNA, allowing for the prophage to be stably maintained within the host genome.

Submit
85. A researcher finds that Streptococcus pneumoniae injected intraperitoneally is uniformly fatal to mice. Antibodies with which of the following in vitro effects on the bacterium would prevent this outcome?

Explanation

Capsular swelling refers to the enlargement of the capsule surrounding the bacterium. Streptococcus pneumoniae is known to have a capsule that protects it from the host's immune system. If the capsule swells, it becomes thicker and more difficult for the bacterium to evade the immune response. Therefore, antibodies that cause capsular swelling would prevent the bacterium from causing a fatal infection in mice, as it would make it more vulnerable to the immune system's attack.

Submit
86. Gram-positive cocci isolated from the blood of a patient with bacteremia synthesize dextrans from glucose. The bacteria most likely contribute to which of the following pathological states?

Explanation

Gram-positive cocci that synthesize dextrans from glucose are most likely contributing to dental caries. Dextrans are sticky substances that can form a biofilm on the surface of teeth, providing a favorable environment for the growth of bacteria that cause tooth decay. This biofilm, known as dental plaque, can lead to the formation of cavities and other dental problems. Therefore, the presence of these bacteria and their ability to synthesize dextrans suggest a role in the development of dental caries.

Submit
87. A 17-year-old female presents to your office with dark urine and facial puffiness. She had a minor skin infection several weeks ago that resolved spontaneously. Microscopic examination of the urine sediment shows red blood cell casts. The organism responsible for this patient's symptoms would most likely demonstrate:

Explanation

The organism responsible for this patient's symptoms would most likely demonstrate bacitracin sensitivity. This information suggests that the organism is likely to be Streptococcus pyogenes, also known as group A Streptococcus. This bacterium is sensitive to bacitracin, which means it will be inhibited by the antibiotic. The presence of dark urine and facial puffiness, along with red blood cell casts in the urine sediment, suggests a possible diagnosis of glomerulonephritis, which can be caused by a Streptococcus pyogenes infection.

Submit
88. Blood cultures from a 54-year-old male recently diagnosed with Hodgkin's disease reveal motile Gram positive rods that produce a very narrow zone of 13-hemolysis on sheep blood agar. Which of the following processes is most important in eliminating these bacteria from the body?

Explanation

Cell-mediated immunity is the most important process in eliminating the bacteria from the body. This type of immunity involves the activation of T cells, which can directly kill infected cells or activate other immune cells to destroy the bacteria. In the case of the motile Gram positive rods, cell-mediated immunity would be crucial in eliminating these bacteria from the body as it can specifically target and destroy infected cells. Terminal complement cascade, eosinophil action, immunoglobulin secretion, and neutrophil oxidative burst may also play a role in the immune response, but cell-mediated immunity is the most important process in this scenario.

Submit
89. A 45-year-old woman presents to the emergency department with intense pain in her lower back and a burning sensation upon urination. A urine culture was taken and plated on MacConkey agar. Gram-negative rods that did not ferment lactose were identified. Which virulence factor of the causal agent is most important to pathogenesis?

Explanation

Urease is the most important virulence factor for the causal agent in this scenario. Urease is an enzyme that breaks down urea into ammonia and carbon dioxide. This can lead to the formation of alkaline urine, which creates an environment conducive to the growth of certain bacteria, such as Proteus mirabilis. These bacteria are known to cause urinary tract infections and can contribute to the symptoms described by the patient, including intense lower back pain and a burning sensation upon urination.

Submit
90. Roommates of a 19-year-old college student become alarmed when he does not get up to go to swim practice in the morning and they are unable to wake him for his 11 AM class (he had complained of a headache and not feeling well the night before). The rescue squad finds a febrile, comatose young man with a petechial rash. In the emergency room, Kernig and Brudzinski signs are present. No papilledema is seen, so a spinal tap is done. Protein is high, glucose low. CSF WBC count is 9,000 (mainly PMNs) with few RBCs. The characteristics of the most likely causal agent are

Explanation

The characteristics described in the question, such as a febrile, comatose young man with a petechial rash, Kernig and Brudzinski signs, and abnormal CSF findings (high protein, low glucose, high WBC count), are consistent with meningitis. The presence of a Gram-negative, oxidase-positive diplococcus as the most likely causal agent suggests Neisseria meningitidis, which is a common cause of bacterial meningitis in young adults.

Submit
91. Infective endocarditis is known to be caused by many different bacterial species. Which of the following scenarios is most consistent with infective endocarditis caused by Streptococcus bovis?

Explanation

This scenario is most consistent with infective endocarditis caused by Streptococcus bovis because it is well known that this bacterial species is commonly associated with colorectal cancer. The persistent fever in this patient after being diagnosed with colon cancer suggests that the infection may have spread from the colon to the heart, causing infective endocarditis.

Submit
92. A 72-year-old man is brought to the emergency department from a skilled nursing facility because of altered mental status, skin flushing, and a high fever. His blood pressure is 60/20 mmHg, and heart rate is 11 O/min. If blood cultures are positive for E coli, which of the following is the most likely source of bacteremia in this patient?

Explanation

The most likely source of bacteremia in this patient is the urinary tract. The patient's altered mental status, skin flushing, and high fever are consistent with sepsis, and the positive blood cultures for E. coli suggest a urinary tract infection. Urinary tract infections are common in older adults and can lead to bacteremia, especially in patients with underlying comorbidities or residing in skilled nursing facilities. The low blood pressure and low heart rate may be indicative of septic shock, a severe form of sepsis.

Submit
93. Group A Streptococci demonstrate significant resistance to phagocytic killing when placed in fresh human blood. This resistance can be most effectively overcome by adding antibodies to which of the following?

Explanation

Group A Streptococci demonstrate significant resistance to phagocytic killing when placed in fresh human blood. This resistance can be most effectively overcome by adding antibodies to Protein M.

Submit
94. A 65-year-old male hospitalized for community acquired pneumonia develops fever and loose stools on his fourth hospital day. He recently traveled to Mexico for vacation without taking any prophylactic antibiotics or receiving any vaccinations. Sigmoidoscopy demonstrates white/yellow membrane-like plaques on the colonic mucosa and biopsy shows that these plaques are composed of fibrin and inflammatory cells. The patient's condition is most likely associated with which of the following?

Explanation

The patient's condition is most likely associated with antibiotic therapy because the development of fever and loose stools after the fourth hospital day suggests the possibility of antibiotic-associated diarrhea. The presence of white/yellow membrane-like plaques on the colonic mucosa further supports this diagnosis, as it is consistent with pseudomembranous colitis, which is commonly caused by Clostridium difficile infection. This infection is a known complication of antibiotic therapy, as it disrupts the normal gut flora and allows for the overgrowth of C. difficile. The patient's recent travel to Mexico and consumption of home-canned foods are not directly related to the development of his symptoms.

Submit
95. An outbreak of water-born gastroenteritis reported in Latin America is caused by oxidase-positive, gram-negative, comma-shaped rods that can survive on alkaline media. Stool microscopy in these patients is expected to demonstrate:

Explanation

The outbreak of water-born gastroenteritis caused by oxidase-positive, gram-negative, comma-shaped rods indicates an infection with Vibrio cholerae. Stool microscopy in patients with cholera typically shows mucus and some epithelial cells. This is because the bacteria attach to the intestinal epithelial cells, causing them to slough off and appear in the stool. The presence of mucus indicates inflammation and damage to the intestinal lining, which is characteristic of cholera.

Submit
96. A college infirmary has noticed that a number of students have been presenting with similar symptoms, including headache, sensitivity to light, and nausea and vomiting. They were also found to have fevers as high as 40 C ( 104F). Upon examination of these students, a distinctive rash was noted to appear within one day after the onset of the other symptoms. These patients were immediately sent to the nearest emergency department for further workup and treatment. Several of the blood cultures revealed gram-negative kidney bean-shaped cocci. The blood concentration of which of the following microbial components would show the closest correlation with these patients' morbidity and mortality?

Explanation

The presence of gram-negative kidney bean-shaped cocci in the blood cultures suggests that the patients are infected with a gram-negative bacterium. The symptoms described, including the distinctive rash, are consistent with meningococcal meningitis, which is caused by Neisseria meningitidis. Outer membrane lipooligosaccharide (LOS) is a major virulence factor of N. meningitidis and is responsible for the symptoms and complications associated with the infection. LOS induces an inflammatory response and can cause damage to blood vessels, leading to sepsis and meningitis. Therefore, the blood concentration of outer membrane lipooligosaccharide would show the closest correlation with these patients' morbidity and mortality.

Submit
97. In a research experiment, Strain 1 of Streptococcus pneumoniae produces no effects when injected into the mouse peritoneal cavity. Similarly, when a lysate of a virulent strain of S. pneumoniae (Strain 2} is introduced into the peritoneal cavity no effect is observed. However, when Strain 1 is injected with a lysate of Strain 2 death of the mice results. Which of the following processes most likely accounts for the observed phenomenon?

Explanation

The observed phenomenon can most likely be accounted for by chromosomal fragment uptake from media. This is because when Strain 1 is injected with a lysate of Strain 2, the mice die, indicating that some genetic material from Strain 2 is being transferred to Strain 1. Chromosomal fragment uptake from media involves the uptake of DNA fragments from the surrounding environment, which can lead to genetic transfer between bacterial strains. This process would explain why the introduction of Strain 2 lysate has an effect on Strain 1.

Submit
98. An infant presents to the emergency department due to difficulty breath- ing, constipation, and anorexia. Upon examination, the physician notes flaccid paralysis. A toxin screen of the stool identified the agent. What is the mechanism of action of the toxin?

Explanation

The correct answer is "Blocks release of acetylcholine." This toxin is likely causing botulism, which is characterized by flaccid paralysis. Botulinum toxin inhibits the release of acetylcholine at the neuromuscular junction, leading to muscle weakness and paralysis. This is achieved by cleaving SNARE proteins, which are responsible for the fusion of synaptic vesicles containing acetylcholine with the presynaptic membrane. As a result, acetylcholine cannot be released into the synaptic cleft, leading to muscle weakness and paralysis.

Submit
99. A 13-year-old girl presents to her pediatrician with fever, malaise, and a sore throat. Physical examination reveals a fever of 39.4 C (103.0 F), cervical lymphadenopathy, and pharyngeal erythema. A swab is taken from some of the tonsillar exudate and cultured on blood agar. Culture reveals beta hemolytic, gram-positive cocci, and a rapid antigen test is positive. What is the major component that protects the causal agent from osmotic damage?

Explanation

Peptidoglycan is the major component that protects the causal agent from osmotic damage. Peptidoglycan is a structural component of the cell wall in gram-positive bacteria, which includes the causal agent in this case. It provides rigidity and strength to the cell wall, preventing it from bursting or collapsing due to changes in osmotic pressure. This is particularly important in gram-positive bacteria because they lack an outer membrane like gram-negative bacteria, making them more susceptible to osmotic damage.

Submit
100. Several postal workers come down with symptoms of dyspnea, cyano- sis, hemoptysis, and chest pain. Chest x-ray reveals mediastinal widen- ing. Sputum cultures are negative for all routine respiratory pathogens. Serology correctly identifies the causal agent. Which of the following struc- tures is possessed by the causal agent?

Explanation

The causal agent possesses a spore.

Submit
101. A group of physicians in a developing country is advocating the early empiric use of penicillin in the treatment of clinically diagnosed bacterial pharyngitis, a condition that is quite common in children of that nation. What would you expect to decrease as a result of their policy in the long term?

Explanation

The early empiric use of penicillin in the treatment of bacterial pharyngitis can help prevent the development of rheumatic heart disease, a condition that can lead to the need for cardiac surgeries. By treating bacterial pharyngitis promptly with penicillin, the physicians can reduce the incidence of rheumatic heart disease, thus decreasing the need for cardiac surgeries in the long term.

Submit
102. A 6-year-old immigrant from Eastern Europe is brought to the emergency department by his parents after they found him having difficulty breathing. The child has a fever and has not been eating or drinking for the last several hours. Head and neck examination reveals neck swelling, palatal paralysis, and a gray pharyngeal exudate. The parents are unable to provide information regarding the child's vaccination history. Which of the following interventions is most likely to improve this patient's prognosis?

Explanation

Passive immunization is the most likely intervention to improve this patient's prognosis. The presentation of fever, difficulty breathing, neck swelling, palatal paralysis, and gray pharyngeal exudate suggests a possible diagnosis of diphtheria. Diphtheria is caused by the bacterium Corynebacterium diphtheriae, which produces a toxin that can lead to severe respiratory distress and other complications. Passive immunization with diphtheria antitoxin can provide immediate protection against the toxin and improve the patient's prognosis. Antibiotic therapy may also be necessary to eradicate the bacteria, but passive immunization is the most critical intervention in this case.

Submit
103. Gram stain of a cerebrospinal fluid (CSF) sample from a 21 -year-old Caucasian male army recruit with fever and headache demonstrates bean-shaped Gram negative cocci in pairs. Which of the following is the most likely route of meningeal contamination in this patient?

Explanation

The most likely route of meningeal contamination in this patient is through the pharynx, blood, choroid plexus, and then the meninges. This is because the Gram stain of the cerebrospinal fluid sample showed bean-shaped Gram-negative cocci in pairs, which is indicative of Neisseria meningitidis, a common cause of bacterial meningitis. Neisseria meningitidis is typically found in the pharynx and can enter the bloodstream, allowing it to reach the meninges. The choroid plexus is a network of blood vessels in the brain that produces cerebrospinal fluid, so it is possible for the bacteria to contaminate the CSF through this route.

Submit
104. There is a specific bacterial product that, when injected locally into the muscles of patients with relentless focal dystonias such as torticollis, produces a dramatic but temporary relief of symptoms. This substance is produced by bacteria that demonstrate:

Explanation

Subterminal spore formation refers to the production of spores by bacteria at the end of their cells. These spores are highly resistant to harsh conditions and can survive for long periods of time. In the given scenario, the bacterial product injected into the muscles of patients with dystonias produces temporary relief of symptoms. The presence of subterminal spore formation suggests that the bacteria are capable of surviving in adverse conditions within the patient's muscles, allowing them to continue producing the beneficial product.

Submit
105. A 74-year-old previously healthy Caucasian male comes to his physician's office complaining of abrupt onset fever, headache, myalgias, malaise, cough and throat pain. His two granddaughters missed several days of school because of similar symptoms. Examination demonstrates mild hyperemia of the throat without any exudate, and the patient is sent home on conservative management. Five days later, he is admitted to the hospital with progressive dyspnea, chest pain, and productive cough. Which of the following pathogens is most likely to be isolated from this patient's sputum?

Explanation

The patient's symptoms of fever, headache, myalgias, malaise, cough, and throat pain are consistent with a respiratory infection. The progression to dyspnea, chest pain, and productive cough suggests a worsening of the infection. Staphylococcus aureus is a common cause of respiratory infections and can lead to severe pneumonia. Additionally, the fact that the patient's granddaughters had similar symptoms suggests a possible transmission of the pathogen within the family. Therefore, Staphylococcus aureus is the most likely pathogen to be isolated from the patient's sputum.

Submit
106. A 23-year-old male is being evaluated for persistent cough and a pulmonary infiltrate detected on chest Xray. 3 ml of this patient's blood is added to an anticoagulated tube and placed into iced water. Several minutes later clumping is detected inside the tube, but it rapidly uncoagulates when the tube is warmed while being held it in the observer's hand. Which of the following organisms is most likely responsible for this patient's condition?

Explanation

The clumping of blood in the anticoagulated tube followed by rapid uncoagulation when warmed suggests the presence of cold agglutinins, which are antibodies that can cause agglutination of red blood cells at low temperatures. Mycoplasma pneumoniae is known to be associated with cold agglutinin disease, a condition characterized by the production of these antibodies. Therefore, Mycoplasma pneumoniae is the most likely organism responsible for this patient's condition.

Submit
107. A 5-year-old boy is found to have a high serum level of antibodies against polyribitol ribose phosphate (PRP). The antibodies would most likely carry protection against:

Explanation

Epiglottitis is a bacterial infection caused by Haemophilus influenzae type b (Hib), which is known to have a capsule composed of polyribitol ribose phosphate (PRP). Antibodies against PRP provide protection against Hib infection, including epiglottitis. Therefore, the high serum level of antibodies against PRP in the 5-year-old boy suggests that he is protected against epiglottitis.

Submit
108. A 70-year-old man presents to the emergency department with a fever of 39.7 C (103.5 F), a dry cough, tachypnea, and chest pain. History reveals he has been smoking since he was a teen. He mentions that several people at the assisted living community where he resides have had similar symptoms. A sputum sample isolated organisms that grew on buffered charcoal yeast extract agar and stained weakly gram-negative. Which of the following properties is consistent with the organism?

Explanation

The correct answer is "Requires iron and cysteine for growth." This property is consistent with Legionella pneumophila, the organism causing Legionnaires' disease. Legionella pneumophila is an intracellular gram-negative bacterium that requires iron and cysteine for growth. It is commonly associated with water sources such as cooling towers, hot tubs, and plumbing systems, and can cause severe pneumonia in susceptible individuals, especially those with a history of smoking.

Submit
109. A 68-year-old woman on chemotherapy for leukemia has developed sepsis due to an infection with Escherichia coli. The following day the patient devel- ops septic shock and dies. The structure on the bacterium most likely respon- sible for causing septic shock in this patient is

Explanation

Lipopolysaccharide (LPS) is a component of the outer membrane of Gram-negative bacteria like Escherichia coli. It is a potent endotoxin that can trigger a severe immune response in the body, leading to septic shock. In this case, the patient developed sepsis due to an E. coli infection, and the subsequent development of septic shock suggests that LPS from the bacteria played a role in causing the shock. Therefore, lipopolysaccharide is the most likely structure on the bacterium responsible for causing septic shock in this patient.

Submit
110. Healthy volunteers orally inoculated with pathogenic strains of Clostridium difficile do not develop signs of infection. Which of the following protective mechanisms is most likely responsible?

Explanation

Intestinal biomass refers to the total mass of microorganisms present in the intestine. These microorganisms play a crucial role in maintaining the balance of the gut microbiota and preventing the colonization of pathogenic bacteria like Clostridium difficile. Therefore, a high intestinal biomass can help prevent the development of infection by inhibiting the growth and colonization of pathogenic strains. This is why healthy volunteers orally inoculated with pathogenic strains of Clostridium difficile do not develop signs of infection.

Submit
111. A 1-year-old boy is brought to the emergency department by his mother because of high fever, irritability, and photophobia. He recently underwent ventriculoperitoneal shunt placement for congenital hydrocephalus. On physical examination, his temperature is 38.3° C (101° F), and passive flexion of the neck results in spontaneous flexion of the hips and knees. Blood cultures grow coagulase-negative Staphylococcus. Which of the following is the most important virulence mechanism by which this bacterium causes disease?

Explanation

The correct answer is "Synthesis of an extracellular polysaccharide matrix". Coagulase-negative Staphylococcus, commonly found on the skin, can cause infections in patients with indwelling medical devices, such as ventriculoperitoneal shunts. The bacteria are able to adhere to the surface of the device and form a biofilm, which is a polysaccharide matrix that protects them from the host immune response and antibiotics. This biofilm formation is a key virulence mechanism that allows the bacteria to persist and cause disease in these patients.

Submit
112. A community hospital is experiencing an increased incidence of nosocomial pneumonias. Most cases occur in patients with long hospitalizations. The microorganism isolated in several cases is visualized with silver stains and demonstrates slow growth on complex media such as charcoal-yeast extract supplemented with cysteine. Which of the following is most likely implicated in the increased incidence of nosocomial pneumonias in the hospital?

Explanation

The correct answer is colonization of the hospital water system. This is because the microorganism isolated in the cases is visualized with silver stains and demonstrates slow growth on complex media, indicating that it is likely a waterborne pathogen. The increased incidence of nosocomial pneumonias in patients with long hospitalizations suggests that they are being exposed to the pathogen over an extended period of time, possibly through contaminated water sources within the hospital.

Submit
113. A 24-year-old African-American female presents to the emergency room with fever, malaise, and intense pain over her right thigh. She has a long history of frequent presentation to the hospital with painful crises. Usually she is admitted and treated with supplemental oxygen and intravenous narcotics. This episode of pain is very different because she is febrile (103.0° F) with exquisite tenderness over the right thigh, as well as local warmth. You send her for an MRI, and based on those results, you wish to begin antibiotics promptly. Which of the following organisms is most likely causing her clinical presentation?

Explanation

The patient's presentation with fever, malaise, intense pain over the right thigh, and local warmth suggests an acute infection. The fact that she has a long history of painful crises indicates a chronic condition, most likely sickle cell disease. Patients with sickle cell disease are at increased risk for infections, particularly by encapsulated organisms such as Salmonella. Therefore, Salmonella is the most likely organism causing her clinical presentation.

Submit
114. An 8-year-old male who recently moved to the United States from Asia is brought to the emergency room with throat pain and difficulty breathing. Soon after being admitted to the hospital, he dies of severe heart failure. In the laboratory, bacterial isolates from this patient's tonsils are found to cause rapid death of 8 out of 1 O exposed guinea pigs. Two of the 10 experimental animals survive. The surviving animals most likely possess which of the following antibodies?

Explanation

The correct answer is "lgG against circulating proteins." This is because the patient's death was caused by severe heart failure, which suggests that the bacteria in his tonsils were able to enter his bloodstream and cause systemic infection. The fact that the bacterial isolates caused rapid death in guinea pigs also indicates that they were able to circulate throughout the body and cause widespread damage. lgG antibodies are the most abundant type of antibody in the bloodstream and are responsible for neutralizing and eliminating circulating pathogens, making them the most likely antibodies present in the surviving animals.

Submit
115. A 59-year-old female is brought to the ER with fever, skin flushing, and an altered level of consciousness. Her blood pressure is 50/20 mmHg, and her heart rate is 120/min. If blood cultures are positive for E coli, which of the following bacterial factors is most likely responsible for this patient's current condition?

Explanation

Lipid A is the lipid component of lipopolysaccharide (LPS), which is found in the outer membrane of Gram-negative bacteria like E. coli. It is a potent endotoxin that can cause a severe immune response in the host. In this case, the patient's symptoms of fever, skin flushing, and altered level of consciousness, along with low blood pressure and high heart rate, are consistent with septic shock. Lipid A is known to trigger the release of inflammatory mediators, leading to the systemic inflammatory response seen in septic shock. Therefore, Lipid A is most likely responsible for this patient's current condition.

Submit
116. A 65-year-old male taking amoxicillin for sinusitis is hospitalized with diarrhea and abdominal cramps. Complete blood count shows leukocytosis. This toxin responsible for his current condition primarily damages which of the following components of intestinal mucosal cells?

Explanation

The correct answer is cytoskeleton integrity. The patient's symptoms of diarrhea and abdominal cramps suggest damage to the intestinal mucosal cells. The cytoskeleton is responsible for maintaining the structural integrity of cells and plays a crucial role in cell movement and shape. Damage to the cytoskeleton can disrupt the normal functioning of intestinal cells, leading to symptoms such as diarrhea and abdominal cramps.

Submit
117. A developing country is experiencing a rise in the incidence of neonatal tetanus (first 28 days after the birth). Which of the following measures would be the most appropriate prevention measure to control the problem?

Explanation

Vaccination of young adults would be the most appropriate prevention measure to control the rise in neonatal tetanus in a developing country. This is because neonatal tetanus is often caused by unhygienic practices during childbirth, such as cutting the umbilical cord with unsterile instruments. By vaccinating young adults, who are often the ones delivering the babies, they can be protected against tetanus and reduce the risk of transmitting the infection to newborns. This measure helps to break the cycle of transmission and prevent neonatal tetanus cases from occurring.

Submit
118. A 52-year-old Asian male presents to your office with cough, night sweats and occasional hemoptysis. Sputum cultures placed on a selective medium grow mycobacteria microscopically observed to grow in parallel chains ('serpentine cords"). This observed bacterial growth pattern most strongly correlates with:

Explanation

The observed bacterial growth pattern of mycobacteria growing in parallel chains, also known as "serpentine cords," is most strongly correlated with virulence. Virulence refers to the ability of a pathogen to cause disease in a host. The formation of serpentine cords by mycobacteria is a characteristic feature of highly virulent strains, particularly Mycobacterium tuberculosis. This growth pattern allows the bacteria to evade immune defenses and establish a persistent infection within the host. Therefore, the presence of serpentine cords in sputum cultures suggests a higher likelihood of a virulent mycobacterial infection.

Submit
119. A 2-day-old newborn develops lethargy and respiratory distress. Blood cultures grow beta-hemolytic Gram-positive cocci in chains that are bacitracin-resistant. Which of the following measures could have prevented this patient's condition most effectively?

Explanation

Intrapartum ampicillin is the most effective measure to prevent the condition described in the question. The patient in the question is presenting with symptoms of early-onset neonatal sepsis, which is commonly caused by Group B Streptococcus (GBS) infection. Intrapartum ampicillin is the recommended treatment for GBS colonization in pregnant women to prevent transmission of the infection to the newborn. Penicillin at 30 weeks gestation is not effective as it is given too early in the pregnancy. Maternal vaccination may help prevent certain infections, but it is not specifically targeted towards GBS. Postnatal immunoglobulin and breastfeeding restriction do not directly address the prevention of GBS infection during childbirth.

Submit
120. A 34-year-old HIV-positive man comes to the emergency department with sudden-onset fever, chills, productive cough, and left-sided chest pain that worsens with deep breathing. His symptoms began 3 days ago. Physical examination shows bronchial breath sounds over the left lower lung. His most recent CD4+ lymphocyte count measured 1 month ago was 800 cellslµL. Which of the following organisms is most likely responsible for this patient's symptoms?

Explanation

The patient's symptoms of sudden-onset fever, chills, productive cough, and left-sided chest pain that worsens with deep breathing, along with bronchial breath sounds over the left lower lung, are indicative of pneumonia. Streptococcus pneumoniae is the most common cause of community-acquired pneumonia, especially in patients with HIV. Additionally, the patient's recent CD4+ lymphocyte count of 800 cells/μL suggests that his immune system is compromised, making him more susceptible to infections, including pneumonia caused by Streptococcus pneumoniae.

Submit
121. An 8-year-old boy presents to the emergency department with vomiting and a severe cough in which he can't catch his breath. His vaccination his- tory is incomplete. Physical exam reveals fever and conjunctival injection. A nasopharyngeal aspirate grew gram-negative coccobacilli on Bordet- Gengou media. What is the mechanism of action of the toxin involved?

Explanation

The correct answer is ADP ribosylation of Gi. ADP ribosylation is a process in which ADP-ribose is transferred from NAD+ to a target protein, modifying its function. In this case, the toxin involved in the patient's condition is causing ADP ribosylation of the G protein subunit Gi. This modification disrupts the normal function of Gi, which is involved in inhibiting the production of cyclic AMP (cAMP). As a result, cAMP levels increase, leading to excessive fluid secretion and inflammation in the respiratory tract, causing the symptoms observed in the patient.

Submit
122. A 30-year-old man presents to his physician with complaints of midepi- gastric pain. He describes the pain as moderate, occasionally waking him at night, and improving immediately following meals. A urease breath test was positive. Which of the following correctly describes the causal agent?

Explanation

The correct answer describes the causal agent as a gram-negative curved rod that is microaerophilic. This is likely referring to Helicobacter pylori, a bacterium known to cause gastric ulcers and gastritis. H. pylori has a characteristic curved shape and is able to survive in environments with low oxygen levels. The positive urease breath test is a diagnostic test used to detect the presence of H. pylori in the stomach.

Submit
123. A 35-year-old man who is positive for HIV develops sepsis with the sub- sequent development of a necrotic lesion on the buttock that has a black center and an erythematous margin. Which of the following is the most likely causal agent?

Explanation

Pseudomonas aeruginosa is the most likely causal agent in this case because it is known to cause necrotizing soft tissue infections, especially in immunocompromised individuals like this patient who is positive for HIV. The black center and erythematous margin of the lesion are consistent with the characteristic appearance of a necrotic lesion caused by Pseudomonas aeruginosa. Bacillus anthracis, Clostridium perfringens, Enterococcus faecalis, and Staphylococcus aureus are not typically associated with this presentation.

Submit
124. Transient bacteremia is demonstrated in a healthy 22-year-old male after a dental procedure. The Gram-positive bacteria isolated from the blood synthesize dextrans from sucrose. Which of the following would be the most likely adherence site for these bacteria?

Explanation

Fibrin-platelet aggregates would be the most likely adherence site for the bacteria in this case. Transient bacteremia refers to the temporary presence of bacteria in the bloodstream, which can occur after certain procedures like dental work. The fact that the bacteria isolated from the blood synthesize dextrans from sucrose suggests that they have the ability to form biofilms. Fibrin-platelet aggregates are commonly found in areas of vascular injury or inflammation, making them a favorable site for bacterial adherence and biofilm formation.

Submit
125. A 34-year-old woman comes to the physician complaining of fever, dysuria, and flank pain. Her temperature is 37.8 C (100 F), blood pressure is 122/78 mm Hg, and pulse is 84/min. Physical examination elicits suprapubic and costovertebral angle tenderness. Urine dipstick analysis reveals positive leukocyte esterase and nitrite. A subsequent urine culture grows colonies of motile Gram-negative rods demonstrating a green metallic sheen on eosin methylene blue (EMB} agar and hemolysis on blood agar. Which of the following is the most important bacterial factor responsible for this patient's condition?

Explanation

The most important bacterial factor responsible for this patient's condition is the fimbrial antigen. Fimbriae are hair-like structures found on the surface of bacteria, and they play a crucial role in the attachment of bacteria to host cells. In this case, the presence of fimbrial antigen suggests that the bacteria causing the infection have the ability to adhere to the urinary tract epithelium, leading to symptoms such as dysuria and suprapubic tenderness. Additionally, the positive leukocyte esterase and nitrite on urine dipstick analysis indicate the presence of a urinary tract infection, further supporting the role of the fimbrial antigen in this patient's condition.

Submit
126. A 34-year-old man is admitted to the intensive care unit because of fever, chills, shortness of breath, and altered mental status. He has had these symptoms for the past one week but has worsened over the last 24 hours. His past medical history is significant for a motor vehicle accident two years ago in which he sustained blunt abdominal trauma and required emergent laparotomy. His blood pressure is 80/40 mm Hg and pulse is 120/min. He is initiated on broad spectrum antibiotics, intravenous fluids, and vasopressors. Despite extensive resuscitative attempts, he dies 2 hours later. Blood cultures obtained on admission grow Streptococcus pneumoniae. Impairment of which of the following protective mechanisms most likely contributed to the severity of this patient's infection?

Explanation

The impairment of bacterial clearance most likely contributed to the severity of this patient's infection. Bacterial clearance refers to the body's ability to remove bacteria from the bloodstream and tissues. In this case, the patient's symptoms of fever, chills, shortness of breath, and altered mental status suggest a severe infection. The fact that blood cultures obtained on admission grew Streptococcus pneumoniae further supports the presence of a bacterial infection. The patient's history of a motor vehicle accident and blunt abdominal trauma may have compromised the body's ability to clear the bacteria, leading to the progression of the infection and ultimately, the patient's death.

Submit
127. Neurotransmitter release from certain spinal cord inhibitory interneurons inhibits polysynaptic spread of impulses and insures proper functioning of agonist and antagonist skeletal muscles. Absence of agonist/antagonist muscle coordination and sustained impulse spread leading to spasm can result from inadequate release of:

Explanation

The correct answer is Glycine due to C. tetani toxin. This is because the question states that neurotransmitter release from certain inhibitory interneurons in the spinal cord inhibits the spread of impulses and ensures proper functioning of agonist and antagonist skeletal muscles. The absence of agonist/antagonist muscle coordination and sustained impulse spread leading to spasm can result from inadequate release of glycine. C. tetani toxin specifically targets and inhibits the release of glycine, leading to muscle spasms and the condition known as tetanus.

Submit
128. A 65-year-old male presents to the ER with a sore on his right hand. He is involved in the wool business and has recently returned from a tour of wool processing plants in a series of Asian countries. Physical examination reveals an ulcer with black eschar and significant surrounding edema. The toxin that caused edema around the ulcer in this patient has an action most similar to that of a toxin produced by:

Explanation

The correct answer is Bordetella pertussis. Bordetella pertussis produces a toxin called pertussis toxin, which causes edema around the ulcer. This toxin is similar in action to the toxin produced by the bacteria causing the patient's symptoms. Streptococcus pyogenes causes streptococcal toxic shock syndrome, which does not typically present with a black eschar. Shigella dysenteriae causes dysentery and does not typically cause ulcers with black eschar. Clostridium botulinum produces botulinum toxin, which causes muscle paralysis and does not typically cause ulcers. Clostridium difficile causes pseudomembranous colitis and does not typically cause ulcers with black eschar.

Submit
129. A patient with a non-healing skin lesion has that lesion biopsied to deter- mine its cause. The pathology lab reports back that the lesion has the char- acteristics of a stellate granuloma. Which of the following is most likely to be true of the causal agent?

Explanation

The causal agent of the non-healing skin lesion being intracellular means that it is located inside the cells. This suggests that the agent may be a bacterium or a virus that is able to invade and replicate within the host cells. This can hinder the healing process and make it difficult to treat the lesion effectively.

Submit
130. A 4-year-old boy develops several honey-crusted lesions behind his ears and on his face. The simplest test for the physician to determine the genus of bacteria responsible for this child's illness is the

Explanation

The correct answer is the Catalase test. This test is used to determine whether a bacterium produces the enzyme catalase, which breaks down hydrogen peroxide into water and oxygen. Staphylococcus aureus, the bacterium responsible for causing impetigo, a skin infection characterized by honey-crusted lesions, produces catalase. Therefore, performing the catalase test can help identify the genus of bacteria responsible for the child's illness. The other options listed are not specific to identifying the genus of bacteria causing the illness.

Submit
131. A 27-year-old woman, after returning home from her honeymoon, has developed urinary frequency, dysuria, and urgency. Her urine is grossly bloody. Which lab data are most likely to define the causal agent?

Explanation

The symptoms described in the question, including urinary frequency, dysuria, urgency, and grossly bloody urine, are consistent with a urinary tract infection (UTI). The most likely causal agent of a UTI is a gram-negative bacterium. The ability to reduce nitrates to nitrites is a characteristic of certain gram-negative bacteria, such as Escherichia coli, which is the most common cause of UTIs. Therefore, the lab data that would define the causal agent in this case is a gram-negative bacterium capable of reducing nitrates to nitrites.

Submit
132. A 63-year-old man is brought to the emergency department after recent onset of high fever, confusion, headache, watery diarrhea, and a mildly productive cough. He has been smoking two packs of cigarettes daily for more than 30 years and has been diagnosed with chronic bronchitis. Otherwise, his past medical history is not significant. His temperature is 40.1 C (104 F), blood pressure is 100/70 mm Hg, pulse is 91 /min, and respirations are 28/min. Sputum Gram staining reveals numerous neutrophils but no bacteria. Which of the following is the most likely cause of this patient's disease?

Explanation

The patient's symptoms of high fever, confusion, headache, watery diarrhea, and mildly productive cough, along with the presence of numerous neutrophils in the sputum Gram staining, suggest a severe and systemic infection. Legionella pneumophila is a gram-negative bacterium that can cause a severe form of pneumonia known as Legionnaires' disease. It commonly affects individuals with risk factors such as smoking and chronic lung diseases like chronic bronchitis. Therefore, Legionella pneumophila is the most likely cause of this patient's disease.

Submit
133. A previously healthy 5-month-old infant presents with apparent upper body weakness including droopy eyes, head lag, drooling, and inability to sit unassisted. The most likely infectious form is

Explanation

The correct answer is endospore. Endospores are a dormant form of bacteria that are highly resistant to environmental stresses such as heat, chemicals, and radiation. They are formed by certain bacteria as a survival mechanism when conditions become unfavorable. In this case, the symptoms described in the infant suggest a neuromuscular weakness, which is not typically associated with elementary bodies, exotoxins, reticulate bodies, or vegetative cells. Therefore, the most likely infectious form causing these symptoms would be an endospore.

Submit
134. A 40-year-old homeless man presents to the emergency department with fever and night sweats, coughing up blood. Acid-fast bacilli are identified in his sputum. Which of the following virulence factors allows the causal agent to inhibit phagosome-lysosome fusion to survive intracellularly?

Explanation

Sulfatides is a virulence factor that allows the causal agent to inhibit phagosome-lysosome fusion to survive intracellularly. This means that the bacteria can prevent the fusion of the phagosome (which engulfs the bacteria) with the lysosome (which contains enzymes that can kill the bacteria). By inhibiting this fusion, the bacteria can avoid being destroyed and can continue to survive and replicate within the host's cells. This ability to evade the host's immune response contributes to the pathogenicity of the bacteria.

Submit
135. A 5-year-old child who recently moved from Eastern Europe is brought to the emergency room with difficulty breathing and a low-grade fever. Physical examination reveals a grayish pharyngeal exudate, enlarged cervical lymph nodes and partial soft palate paralysis. Bacteria isolated from the pharyngeal exudate demonstrate exotoxin production in the laboratory. This exotoxin acts via the following mechanism:

Explanation

The correct answer is "Intracellular protein ribosylation". In this scenario, the child's symptoms, including difficulty breathing, fever, grayish pharyngeal exudate, enlarged cervical lymph nodes, and partial soft palate paralysis, suggest a diagnosis of diphtheria. Diphtheria is caused by the bacteria Corynebacterium diphtheriae, which produces an exotoxin called diphtheria toxin. This toxin acts by ribosylating (adding a ribose sugar) to a specific protein within host cells, inhibiting protein synthesis and causing cell death. This mechanism of intracellular protein ribosylation is responsible for the characteristic symptoms and complications of diphtheria.

Submit
136. A 73-year-old woman comes to the physician complaining of progressive, severe pain and discharge from her left ear for the past 2 days. She has had type 2 diabetes for many years and has been noncompliant with her medications and follow-up appointments. On examination, moving or touching the pinna produces extreme pain. Otoscopic examination shows granulation tissue in the left ear canal with a scant amount of discharge. The tympanic membrane is clear, and there is no middle ear effusion. Initial cultures from the ear show a Gram-negative rod. Which of the following microbiological characteristics best describes the infecting organism?

Explanation

The infecting organism is described as motile and oxidase positive. This characteristic suggests that the organism is capable of movement and produces the enzyme oxidase. This information, along with the clinical presentation of pain and discharge in the ear, suggests that the patient may have an infection caused by Pseudomonas aeruginosa. Pseudomonas aeruginosa is a Gram-negative rod that commonly causes infections in immunocompromised individuals, such as those with poorly controlled diabetes. It is known to be motile and oxidase positive.

Submit
137. A 4-year-old immigrant boy is brought to the pediatric emergency room with a swollen right knee, accompanied by fever and malaise. He is hypotensive and tachycardic. His past medical history is insignificant except for one episode of otitis media. Some of his vaccinations are not up-to-date. Synovial fluid and blood cultures grew pleomorphic gram-negative rods on hematin containing medium. The pathogenicity of the organism responsible for his condition is most likely related to which of the following?

Explanation

The correct answer is Capsule. The presence of pleomorphic gram-negative rods in the synovial fluid and blood cultures suggests a bacterial infection. The fact that the organism grows on hematin-containing medium indicates that it is likely a Haemophilus species. Haemophilus species are known to have a polysaccharide capsule that helps protect them from phagocytosis by the host's immune cells. This capsule is an important virulence factor for these bacteria and allows them to evade the immune system and cause infection.

Submit
138. A 4-year-old boy is brought to the physician by his mother because of decreased appetite and a fever of 100.4° F (38° C) for 2 days. His symptoms have also included abdominal cramps and diarrhea over the last 4 days, which were particularly alarming to the patient's mother when the stools became bloody. Physical examination reveals signs of mild dehydration and a mildly distended abdomen with diffuse pain elicited on palpation. Stool is sent for leukocytes, occult blood, and Gram stain and culture. Fecal leukocytes and occult blood tests are positive, and stool cultures grow non-lactose-fermenting gram-negative rods on MacConkey's agar. The organisms do not produce gas on fermentation of glucose and do not produce H,S on triple sugar iron (TSI) agar. Which of the following mechanisms is the most critical for this bacterium to cause disease?

Explanation

The most critical mechanism for this bacterium to cause disease is mucosal invasion. This is supported by the patient's symptoms of abdominal cramps, diarrhea, and bloody stools, which indicate that the bacterium is able to invade and damage the mucosal lining of the gastrointestinal tract. The positive fecal leukocytes and occult blood tests further suggest an inflammatory response in the mucosa. Other options such as exotoxin production, intestinal colonization, bacteremia, and proliferation in lymph nodes may contribute to the disease process, but mucosal invasion is the primary mechanism responsible for the patient's symptoms.

Submit
139. Three cases of severe pharyngitis were reported in a community of immigrants. The patients had thick pharyngeal exudates, neck swelling, and difficulty swallowing. One of them died from severe heart failure. The toxin responsible has a mechanism of action most similar to another toxin produced by which of the following bacteria?

Explanation

The correct answer is Pseudomonas aeruginosa. Pseudomonas aeruginosa produces a toxin called exotoxin A, which has a mechanism of action similar to the toxin responsible for the severe pharyngitis in the patients. This toxin inhibits protein synthesis by ADP-ribosylation of elongation factor 2, leading to cell death. Staphylococcus aureus produces toxins such as toxic shock syndrome toxin-1 and exfoliative toxins, which have different mechanisms of action. Clostridium difficile produces toxins A and B, which cause pseudomembranous colitis. Clostridium botulinum produces botulinum toxin, which causes botulism. Bordetella pertussis produces pertussis toxin, which causes whooping cough. Vibrio cholerae produces cholera toxin, which causes cholera.

Submit
140. CSF cultures from a neonate with fever and poor feeding reveal motile Gram-negative rods that form pink colonies on Mac Conkey agar. Which of the following is the most important bacterial factor in the development of this infection?

Explanation

The presence of a capsule is the most important bacterial factor in the development of this infection. Capsules are protective layers that surround bacteria and help them evade the immune system. In this case, the motile Gram-negative rods that form pink colonies on Mac Conkey agar are likely to be encapsulated bacteria. The capsule allows the bacteria to resist phagocytosis and other immune responses, increasing their ability to cause infection.

Submit
141. A 15-day-old boy presents with conjunctivitis. Iodine staining bodies are seen in conjunctival scrapings. The most likely infectious form is a(n)

Explanation

The most likely infectious form in this case is an Alcoholic. Alcoholics are at higher risk for developing infections due to their weakened immune system. The conjunctivitis in this 15-day-old boy could be caused by an infection, and the presence of iodine staining bodies in conjunctival scrapings suggests a possible bacterial or fungal infection. Alcoholism can impair the body's ability to fight off infections, making alcoholics more susceptible to various types of infections. Therefore, an alcoholic is the most likely infectious form in this scenario.

Submit
142. A 7-year-old child with a history of recurrent granulomatous skin infections and a prior episode of Aspergillus pneumonia undergoes a partial hepatectomy to treat a poorly draining liver abscess. The patient is thought to have a mutation of a structural component of a neutrophil oxidase. This child also has an increased vulnerability to infections caused by:

Explanation

The patient's history of recurrent granulomatous skin infections and a prior episode of Aspergillus pneumonia suggests an underlying immunodeficiency. The mention of a mutation in a structural component of a neutrophil oxidase indicates a defect in the function of neutrophils, which are an important component of the immune system's defense against bacterial infections. Pseudomonas cepacia is a gram-negative bacterium that can cause severe infections, particularly in individuals with impaired immune function. Therefore, this patient's increased vulnerability to infections caused by Pseudomonas cepacia is consistent with their underlying immunodeficiency.

Submit
143. If one cell of type one (figure below) is mixed into a culture of 100 cells of type two (below), and culture conditions are optimized for conjugation BUT NOT for cell division, the cellular genotype that would predominate after overnight incubation would be that of

Explanation

If the culture conditions are optimized for conjugation but not for cell division, it means that the cells are more likely to undergo conjugation rather than cell division. Conjugation is a process in which genetic material is transferred between cells. In this case, one cell of type one is mixed into a culture of 100 cells of type two. During conjugation, genetic material can be transferred from the donor cell to the recipient cell. Therefore, the most likely outcome is that the recipient cells, which are type two cells, will acquire a new allele from the donor cell. In this case, the new allele is A, so the cellular genotype that would predominate after overnight incubation would be Cell #2 with a new A allele.

Submit
144. A 56-year-old woman who just returned from a cruise trip to Hawaii is brought to the emergency department because of high fever, headache, and a productive cough for the past two days. She also complains of abdominal pain and watery diarrhea. Her other medical problems include non-allergic bronchial asthma and hypertension. She admits to smoking one pack of cigarettes daily for more than 20 years. Her temperature is 40 C (104 F), blood pressure is 94/63 mm Hg, pulse is 90/min, and respirations are 24/min. Lung examination reveals left lower lobe crackles with no wheezing. Her abdomen is soft, non-distended, and non-tender. Chest x-ray shows left lower lobe consolidation. Which of the following additional findings is most likely to be present in this patient?

Explanation

The patient's presentation is consistent with Legionnaires' disease, a severe form of pneumonia caused by the bacterium Legionella pneumophila. Low serum sodium, or hyponatremia, is a common electrolyte abnormality seen in Legionnaires' disease due to the inappropriate release of antidiuretic hormone (ADH) by the infected lung tissue. ADH causes water retention and dilutional hyponatremia. The other answer choices are not associated with Legionnaires' disease.

Submit
145. E co/iisolated from a patient with diarrhea does not ferment sorbitol on sorbitol containing Mac Conkey agar and does not produce glucuronidase. The toxin produced by these bacteria would most likely:

Explanation

The bacteria isolated from the patient with diarrhea do not ferment sorbitol and do not produce glucuronidase. This suggests that the bacteria are likely to be E. coli, specifically a strain known as E. coli O157:H7. This strain produces a toxin called Shiga toxin, which is known to inactivate ribosomal subunits. This toxin disrupts protein synthesis in the host cells by binding to the ribosomes and preventing them from functioning properly. This can lead to cell death and tissue damage, contributing to the symptoms of diarrhea.

Submit
146. A 23-year-old male is being treated for presumed Mycoplasma pneumonia with erythromycin. He demonstrates mild anemia and elevated serum LOH level. Two months later the patient has no anemia. The absence of anemia in this patient at this point is best explained by:

Explanation

The absence of anemia in this patient at this point is best explained by a faded immune response. This suggests that the patient's immune system has successfully cleared the Mycoplasma infection, leading to the resolution of anemia. As the immune response wanes, there is no longer ongoing destruction of red blood cells, resulting in the absence of anemia.

Submit
View My Results

Quiz Review Timeline (Updated): Jan 20, 2023 +

Our quizzes are rigorously reviewed, monitored and continuously updated by our expert board to maintain accuracy, relevance, and timeliness.

  • Current Version
  • Jan 20, 2023
    Quiz Edited by
    ProProfs Editorial Team
  • Dec 29, 2020
    Quiz Created by
    Catherine Halcomb
Cancel
  • All
    All (146)
  • Unanswered
    Unanswered ()
  • Answered
    Answered ()
Eight of 10 family practice residents who had a potluck four days ago...
A 26-year-old male presents to your office with dysuria and urethral...
A 32-year-old migrant ranch worker is brought to the emergency room...
A cancer chemotherapy patient has to have her intravenous port revised...
What is the typical means of transmission of a toxin that blocks the...
A patient is admitted to the hospital because of a bleeding duodenal...
A 24-year-old female presents to your office with burning urination,...
An 18-year-old college freshman is brought to the emergency room with...
A 58-year-old male presents to the ER with fever, headache and...
A 23-year-old Caucasian female is brought to the ER with fever,...
A 22-year-old female presents with severe pain and swelling in her...
A developing country includes infant vaccination with the Haemophi/us...
Bacteria isolated from the lung tissue of a 32-year-old Caucasian male...
A 5-year-old boy is brought to the ER with breathing difficulty,...
A 78-year-old nursing home resident develops high fevers, chest pain...
A sample of contaminated moist soil is heated to 1 oo• C for 15...
A 55-year-old woman had her rheumatic heart valve replaced with a...
Genital examination of a 31-year-old female reveals yellow discharge...
A 24-year-old female presents to your office with burning during...
A 3-year-old male experiences several episodes of otitis media. Middle...
A 45-year-old female executive goes to a cosmetic surgeon with the...
Volunteer studies are used to determine the infective dose of...
A group of investigators is trying to develop an anti-gonococcal...
A 39 year-old paraplegic man with an indwelling bladder catheter...
A 54-year-old male presents to the ER with a sore on his right...
A 67-year-old male is hospitalized with low-grade fevers, fatigue and...
A 29-year-old female presents to clinic with malaise and an extremely...
A 7-year-old Caucasian boy is brought to your office with blisters on...
A 24-year-old male presents to your ER with severe nausea and vomiting...
Clustered Gram positive bacteria are isolated from the tonsillar...
Health authorities are investigating an outbreak of respiratory...
A 7-day-old infant presents to the emergency department with a fever,...
A 15-day-old boy presents with conjunctivitis. Iodine staining bodies...
A 34-year-old woman comes to the physician because of itching around...
A 32-year-old Caucasian male develops profuse watery diarrhea with...
An outbreak of water-borne gastroenteritis with a high mortality is...
A 5-month-old Hispanic boy is brought to the ER with complaints of...
A 12-year-old boy is brought to the physician with fever, chills, and...
Autopsy of a 14-year-old male who died of septic shock shows extensive...
A group of investigators has developed monoclonal antibodies to the...
A county hospital experiences an outbreak of methicillin-resistant...
A 46-year-old female presents to the emergency room with fever, chest...
A 10-year-old Caucasian male is brought to the ER with a several day...
What is the structure that is found in gram-negative but not in gram-...
A 65-year-old man develops pneumonia. The organisms isolated from the...
A 38-year-old man who recently visited India on business presents to...
A 70-year-old man is hospitalized for an infection and treated with...
A 23-year-old Caucasian female presents to clinic with lower abdominal...
A 35-year-old man presents to your office complaining of a hard mass...
Epidemiological analyses show a significant increase in staphylococcal...
A group of researchers is attempting to determine the asymptomatic...
Sixteen residents in a retirement home have fever, malaise, and...
A tourist who recently returned from a trip to Peru goes to her...
A 60-year-old woman is hospitalized following a stroke and develops a...
A 15-year-old girl develops a sore throat, fever, and earache of...
A 62-year-old Caucasian male who recently underwent a mitral valve...
A 52-year-old male is having low-grade fevers after recent replacement...
A 46-year-old male who just returned from a mission trip to Latin...
A 22-year-old male ingests a solution containing "C-labeled urea....
E coli strains isolated from a 4-year-old Caucasian female with bloody...
A 6-year-old male is brought to the pediatric emergency room with...
A 44-year-old man is brought to the emergency department by his wife...
The vaccine that provides protection against H. inffuenzae contains...
 A 5-year-old child of an Eastern European immigrant family is...
A 10-year-old child develops glomerulonephritis a week after he was...
A 10-year-old girl with an incomplete vaccination history presents to...
An 8-year-old Caucasian boy is brought to your office with throat...
A 45-year-old sailor comes to the physician with groin pain and...
A 34-year-old male presents to your office with a painless penile...
An 8-year-old male is brought to the ER with a two day history of...
Non-pathogenic strains of Corynebacterium diphtheriae can acquire...
A 34-year-old male is hospitalized with difficulty swallowing and...
A 32-year-old male presents to the emergency room with progressive...
Gram-positive bacteria are inoculated under the skin of experimental...
Microscopic examination of a sputum sample from a 34-year-old male...
A 35-year-old male hospitalized with extensive trauma following a...
A toxic substance produced by C. perfringens induces massive hemolysis...
A homeless, malnourished chronic alcoholic presents with severe...
A 6-year-old girl had crashed on a toboggan ride and complained of...
A 70-year-old woman is brought to the emergency department by her...
The clinical laboratory reports the presence of 0157:H7 strains of E....
Two days after eating a meal that included home-canned green beans,...
Which genetic material is found in pathogenic Corynebacterium...
How is a prophage created?
A researcher finds that Streptococcus pneumoniae injected...
Gram-positive cocci isolated from the blood of a patient with...
A 17-year-old female presents to your office with dark urine and...
Blood cultures from a 54-year-old male recently diagnosed with...
A 45-year-old woman presents to the emergency department with intense...
Roommates of a 19-year-old college student become alarmed when he does...
Infective endocarditis is known to be caused by many different...
A 72-year-old man is brought to the emergency department from a...
Group A Streptococci demonstrate significant resistance to phagocytic...
A 65-year-old male hospitalized for community acquired pneumonia...
An outbreak of water-born gastroenteritis reported in Latin America is...
A college infirmary has noticed that a number of students have been...
In a research experiment, Strain 1 of Streptococcus pneumoniae...
An infant presents to the emergency department due to difficulty...
A 13-year-old girl presents to her pediatrician with fever, malaise,...
Several postal workers come down with symptoms of dyspnea, cyano- sis,...
A group of physicians in a developing country is advocating the early...
A 6-year-old immigrant from Eastern Europe is brought to the emergency...
Gram stain of a cerebrospinal fluid (CSF) sample from a 21 -year-old...
There is a specific bacterial product that, when injected locally into...
A 74-year-old previously healthy Caucasian male comes to his...
A 23-year-old male is being evaluated for persistent cough and a...
A 5-year-old boy is found to have a high serum level of antibodies...
A 70-year-old man presents to the emergency department with a fever of...
A 68-year-old woman on chemotherapy for leukemia has developed sepsis...
Healthy volunteers orally inoculated with pathogenic strains of...
A 1-year-old boy is brought to the emergency department by his mother...
A community hospital is experiencing an increased incidence of...
A 24-year-old African-American female presents to the emergency room...
An 8-year-old male who recently moved to the United States from Asia...
A 59-year-old female is brought to the ER with fever, skin flushing,...
A 65-year-old male taking amoxicillin for sinusitis is hospitalized...
A developing country is experiencing a rise in the incidence of...
A 52-year-old Asian male presents to your office with cough, night...
A 2-day-old newborn develops lethargy and respiratory distress. Blood...
A 34-year-old HIV-positive man comes to the emergency department with...
An 8-year-old boy presents to the emergency department with vomiting...
A 30-year-old man presents to his physician with complaints of midepi-...
A 35-year-old man who is positive for HIV develops sepsis with the...
Transient bacteremia is demonstrated in a healthy 22-year-old male...
A 34-year-old woman comes to the physician complaining of fever,...
A 34-year-old man is admitted to the intensive care unit because of...
Neurotransmitter release from certain spinal cord inhibitory...
A 65-year-old male presents to the ER with a sore on his right hand....
A patient with a non-healing skin lesion has that lesion biopsied to...
A 4-year-old boy develops several honey-crusted lesions behind his...
A 27-year-old woman, after returning home from her honeymoon, has...
A 63-year-old man is brought to the emergency department after recent...
A previously healthy 5-month-old infant presents with apparent upper...
A 40-year-old homeless man presents to the emergency department with...
A 5-year-old child who recently moved from Eastern Europe is brought...
A 73-year-old woman comes to the physician complaining of progressive,...
A 4-year-old immigrant boy is brought to the pediatric emergency room...
A 4-year-old boy is brought to the physician by his mother because of...
Three cases of severe pharyngitis were reported in a community of...
CSF cultures from a neonate with fever and poor feeding reveal motile...
A 15-day-old boy presents with conjunctivitis. Iodine staining bodies...
A 7-year-old child with a history of recurrent granulomatous skin...
If one cell of type one (figure below) is mixed into a culture of 100...
A 56-year-old woman who just returned from a cruise trip to Hawaii is...
E co/iisolated from a patient with diarrhea does not ferment sorbitol...
A 23-year-old male is being treated for presumed Mycoplasma pneumonia...
Alert!

Advertisement